Re: [obm-l] Particao do Quadrado

2004-03-05 Por tôpico Henrique Patrício Sant'Anna Branco
 A sutileza é que A e B seriam conexos mas não são conexos por caminhos.
 Cada um deles parece uma nuvem de pontos e as componentes conexas
 por caminhos de A e B são pontos. As nuvens são conexas pq qualquer
 função contínua não constante g: R - [0,1]^2 encontra tanto com A
 quanto com B então é impossível fazer uma cisão de A ou B.

Nicolau (ou quem souber responder),

Sei o que é um conjunto conexo por caminhos, mas não sei o que seria um
conjunto (apenas) conexo.
A (união) B, no caso, conteria dois pontos que não pudessem ser ligados por
poligonais? Isso seria um conjunto conexo?

Grato,
Henrique.

=
Instruções para entrar na lista, sair da lista e usar a lista em
http://www.mat.puc-rio.br/~nicolau/olimp/obm-l.html
=


Re: [obm-l] A^2 = I na obm-u de 2003

2004-03-06 Por tôpico Henrique Patrício Sant'Anna Branco
 Aliás este problema usa de novo aquele fato: se a matriz A satisfaz
 p(A) = 0 onde p só tem raízes simples então A é diagonalizável
 (em um corpo que contenha as raízes de p).

Uma curiosidade...
Existe algum outro polinômio p, além do mínimo e do característico, com p(A)
= 0?
Se existe, como achá-lo?

Henrique.

=
Instruções para entrar na lista, sair da lista e usar a lista em
http://www.mat.puc-rio.br/~nicolau/olimp/obm-l.html
=


Re: [obm-l] A^2 = I na obm-u de 2003

2004-03-06 Por tôpico Henrique Patrício Sant'Anna Branco
  Uma curiosidade...
  Existe algum outro polinômio p, além do mínimo e do característico, com
p(A)
  = 0?
  Se existe, como achá-lo?
 
  Henrique.
 
 Qualquer multiplo do polinomio minimo de A tem A como raiz.

Acho que me expressei mal... Queria saber se um polinômio com p(A) = 0, esse
polinômio deveria ser necessariamente o característico ou o mínimo. É
possivel?

Henrique.

=
Instruções para entrar na lista, sair da lista e usar a lista em
http://www.mat.puc-rio.br/~nicolau/olimp/obm-l.html
=


Re: [obm-l] Problema de algelin

2004-03-08 Por tôpico Henrique Patrício Sant'Anna Branco
 Desculpem pela ignorância se isto for algum conceito básico, mas o que vem
a
 ser posto(A)?

Basicamente, é a número máximo de linhas (ou colunas) linearmente
independentes de uma matriz.
Vale notar que o posto segundo linhas e segundo colunas são iguais.

Henrique.

=
Instruções para entrar na lista, sair da lista e usar a lista em
http://www.mat.puc-rio.br/~nicolau/olimp/obm-l.html
=


[obm-l] urgente

2004-03-20 Por tôpico Henrique Lima Santana


  olah pessoal
 por favor, serah q vcs  poderiam  me dar uma ajuda nesses problemas aqui:
1. determinar a capacitancia de um capacitor formado por discos nao 
paralelos,sendo theta o angulo formado pelos eixos dos 2 discos.
2. considere 2 fios carregados com uma corrente i (orientada para cima), de 
modo q eles sejam quase paralelos (ambos de comprimento l). qual a força q o 
fio inclinado exerce no fio reto?
(um fio reto e um quase reto...)
 valeuzão!
  mario cesar

_
MSN Messenger: instale grátis e converse com seus amigos. 
http://messenger.msn.com.br

=
Instruções para entrar na lista, sair da lista e usar a lista em
http://www.mat.puc-rio.br/~nicolau/olimp/obm-l.html
=


Re: [obm-l] funcao geradora de momentos

2004-03-27 Por tôpico Henrique Patrício Sant'Anna Branco
 É faz sentido mas eu não estou conseguindo entender completamente isto.
 A funcao de distruição de probabilidade que voce diz é a density
 function que escrevem os livros ingles?

A função densidade de probabilidade é pra variáveis contínuas. A função
distribuição de probabilidade é sua análoga para variáveis aleatórias
discretas

=
Instruções para entrar na lista, sair da lista e usar a lista em
http://www.mat.puc-rio.br/~nicolau/olimp/obm-l.html
=


Re: [obm-l] funcao geradora de momentos

2004-03-28 Por tôpico Henrique Patrício Sant'Anna Branco
Morgado,

Realmente. Respondi correndo e acabei falando besteira.
Peço desculpas.

Henrique.

- Original Message - 
From: Augusto Cesar de Oliveira Morgado [EMAIL PROTECTED]
To: [EMAIL PROTECTED]
Sent: Saturday, March 27, 2004 9:38 PM
Subject: Re: [obm-l] funcao geradora de momentos


 Henrique,
 não é isso não.
 Dê uma olhada na resposta do Claudio.
 []s
 Morgado

=
Instruções para entrar na lista, sair da lista e usar a lista em
http://www.mat.puc-rio.br/~nicolau/olimp/obm-l.html
=


[obm-l] Limite de duas variáveis

2004-03-31 Por tôpico Henrique Patrício Sant'Anna Branco
Pessoal,

Esse é um problema do meu livro que me deixou intrigado.

Temos a função f(x,y) = arctan(xy)/(xy).
Se 1 - x^2*y/3  f(x,y)  1, o que podemos dizer de limite de f(x,y) quando
(x,y) - (0,0)?

Minha tentativa foi passar os limites nos três membros da inequação:

lim_(x,y)-(0,0) 1 - x^2*y/3 = 1 e lim_(x,y)-(0,0) 1 = 1

Logo 1  lim f(x,y)  1. Na minha interpretação, tal limite não existe, pois
não existe um real L que seja estritamente menor e estritamente maior que 1,
ao mesmo tempo. O problema é que o livro diz que o tal limite é realmente 1.

Como proceder?

Grato,
Henrique.

=
Instruções para entrar na lista, sair da lista e usar a lista em
http://www.mat.puc-rio.br/~nicolau/olimp/obm-l.html
=


Re: [obm-l] Nr. de filhos e filhas

2004-04-08 Por tôpico Eduardo Henrique Leitner
seja U o numero de filhos e A o numero de filhas
do primeiro dado temos que:

U = A + 1

do segundo dado temos que:

U = 2(A - 1)

resolvendo o sistema temos 

A = 3 
U = 4

On Thu, Apr 08, 2004 at 09:50:06PM -0300, Jerry Eduardo wrote:
 Alguem pode me ajudar a resolver o exercicio abaixo?
 
 Um casal tem filhos e filhas. Cada filho tem o numero de 
 irmaos igual ao numero de irmas. Cada filha tem o numero de 
 irmaos igual ao dobro do numero de irmas. 
 Qual eh o total de filhos e filhas do casal?
 
 Obrigado,
 
 []'s,
 
 Jerry
=
Instruções para entrar na lista, sair da lista e usar a lista em
http://www.mat.puc-rio.br/~nicolau/olimp/obm-l.html
=


[obm-l] Problema de Cálculo

2004-04-16 Por tôpico Henrique Patrício Sant'Anna Branco
Pessoal, este tá me dando dor de cabeça. Alguém pode me dar um caminho?

Suponha que a temperatura em graus Celsius em um ponto (x, y) no plano xy
seja T(x, y) = x*sen(2y) e que a distância no plano xy seja medida em
metros. Uma partícula está se movendo no sentido horário ao redor da
circunferência de raio 1m, centrada na orgem a uma taxa constante de 2 m/s.

a) Qual a velocidade da variação de temperatura 'sentida' pela partícula em
graus Celius por metro, no ponto P(1/2, sqrt(3)/2)?

b) Qual é a velocidade da variação de temperatura sentida pela partícula em
graus Celsius por segundo em P?

Pensei em parametrizar a função pro círculo, colocando x = cos(t) e y =
sen(t). Mas tenho muita idéia do que fazer com a velocidade da partícula...
Agradeço qualquer ajuda.

Henrique.

=
Instruções para entrar na lista, sair da lista e usar a lista em
http://www.mat.puc-rio.br/~nicolau/olimp/obm-l.html
=


[obm-l] Re: [obm-l] dúvi da

2004-04-16 Por tôpico Henrique Patrício Sant'Anna Branco
Fabiano,

Você sabe que ln(x) = int (1/t) dt, t = 0 .. x, certo?
Talvez seja por isso... Mas alguém pode dar uma opinião mais precisa (ou
certa).

Abraços,
Henrique.

- Original Message - 
From: Fabiano Sant'Ana [EMAIL PROTECTED]
To: [EMAIL PROTECTED]
Sent: Friday, April 16, 2004 4:42 PM
Subject: [obm-l] Re: [obm-l] Re: [obm-l] Re: [obm-l] Re: [obm-l] Re: [obm-l]
dúvi da


 por que Natural?
 - Original Message -
 From: Artur Costa Steiner [EMAIL PROTECTED]
 To: [EMAIL PROTECTED]
 Sent: Friday, April 16, 2004 4:17 PM
 Subject: [obm-l] Re: [obm-l] Re: [obm-l] Re: [obm-l] Re: [obm-l] dúvi da


  ln (com um n soh) significa logaritmo neperiano (em homenagem a Napier)
e
 eh
  a mesma coisa que logaritmo na base e = 2,718281828459045., a
 constante
  de Euler. O logaritmo neperiano eh tambem conhecido por logarimo
natural,
  embora nem todos o considerem natural.
  Artur
 
 
  - Mensagem Original 
  De: [EMAIL PROTECTED]
  Para: [EMAIL PROTECTED] [EMAIL PROTECTED]
  Assunto: [obm-l] Re: [obm-l] Re: [obm-l] Re:_[obm-l]_dúvi da
  Data: 16/04/04 14:47
 
  Hm...
  eu SEMPRE pensei que lnn fosse logaritmo neperiano (Algo assim) e que
  logaritmo neperiano fosse log na Base e...
  alguem pode me esclarecer quanto a isso?
 
  fabiano sant'ana
  - Original Message -
  From: Nicolau C. Saldanha [EMAIL PROTECTED]
  To: [EMAIL PROTECTED]
  Sent: Friday, April 16, 2004 1:50 PM
  Subject: [obm-l] Re: [obm-l] Re:_[obm-l]_dúvida
 
 
  ---
  Outgoing mail is certified Virus Free.
  Checked by AVG anti-virus system (http://www.grisoft.com).
  Version: 6.0.655 / Virus Database: 420 - Release Date: 08/04/04
 
 
=
  Instruções para entrar na lista, sair da lista e usar a lista em
  http://www.mat.puc-rio.br/~nicolau/olimp/obm-l.html
 
=
 
  
  OPEN Internet
  @ Primeiro provedor do DF com anti-vírus no servidor de e-mails @
 
 
 
=
  Instruções para entrar na lista, sair da lista e usar a lista em
  http://www.mat.puc-rio.br/~nicolau/olimp/obm-l.html
 
=
 

 =
 Instruções para entrar na lista, sair da lista e usar a lista em
 http://www.mat.puc-rio.br/~nicolau/olimp/obm-l.html
 =


=
Instruções para entrar na lista, sair da lista e usar a lista em
http://www.mat.puc-rio.br/~nicolau/olimp/obm-l.html
=


[obm-l] Re: [obm-l] dúvi da

2004-04-16 Por tôpico Henrique Patrício Sant'Anna Branco
Na verdade, os limites de integração são 1 e x...
Sorry pelo lapso.

Henrique.

- Original Message - 
From: Henrique Patrício Sant'Anna Branco [EMAIL PROTECTED]
To: [EMAIL PROTECTED]
Sent: Friday, April 16, 2004 5:44 PM
Subject: Re: [obm-l] dúvi da


 Fabiano,

 Você sabe que ln(x) = int (1/t) dt, t = 0 .. x, certo?
 Talvez seja por isso... Mas alguém pode dar uma opinião mais precisa (ou
 certa).

 Abraços,
 Henrique.

 - Original Message - 
 From: Fabiano Sant'Ana [EMAIL PROTECTED]
 To: [EMAIL PROTECTED]
 Sent: Friday, April 16, 2004 4:42 PM
 Subject: [obm-l] Re: [obm-l] Re: [obm-l] Re: [obm-l] Re: [obm-l] Re:
[obm-l]
 dúvi da


  por que Natural?
  - Original Message -
  From: Artur Costa Steiner [EMAIL PROTECTED]
  To: [EMAIL PROTECTED]
  Sent: Friday, April 16, 2004 4:17 PM
  Subject: [obm-l] Re: [obm-l] Re: [obm-l] Re: [obm-l] Re: [obm-l] dúvi da
 
 
   ln (com um n soh) significa logaritmo neperiano (em homenagem a
Napier)
 e
  eh
   a mesma coisa que logaritmo na base e = 2,718281828459045., a
  constante
   de Euler. O logaritmo neperiano eh tambem conhecido por logarimo
 natural,
   embora nem todos o considerem natural.
   Artur
  
  
   - Mensagem Original 
   De: [EMAIL PROTECTED]
   Para: [EMAIL PROTECTED] [EMAIL PROTECTED]
   Assunto: [obm-l] Re: [obm-l] Re: [obm-l] Re:_[obm-l]_dúvi da
   Data: 16/04/04 14:47
  
   Hm...
   eu SEMPRE pensei que lnn fosse logaritmo neperiano (Algo assim) e que
   logaritmo neperiano fosse log na Base e...
   alguem pode me esclarecer quanto a isso?
  
   fabiano sant'ana
   - Original Message -
   From: Nicolau C. Saldanha [EMAIL PROTECTED]
   To: [EMAIL PROTECTED]
   Sent: Friday, April 16, 2004 1:50 PM
   Subject: [obm-l] Re: [obm-l] Re:_[obm-l]_dúvida
  
  
   ---
   Outgoing mail is certified Virus Free.
   Checked by AVG anti-virus system (http://www.grisoft.com).
   Version: 6.0.655 / Virus Database: 420 - Release Date: 08/04/04
  
  
 =
   Instruções para entrar na lista, sair da lista e usar a lista em
   http://www.mat.puc-rio.br/~nicolau/olimp/obm-l.html
  
 =
  
   
   OPEN Internet
   @ Primeiro provedor do DF com anti-vírus no servidor de e-mails @
  
  
  
 =
   Instruções para entrar na lista, sair da lista e usar a lista em
   http://www.mat.puc-rio.br/~nicolau/olimp/obm-l.html
  
 =
  
 
 
=
  Instruções para entrar na lista, sair da lista e usar a lista em
  http://www.mat.puc-rio.br/~nicolau/olimp/obm-l.html
 
=
 


=
Instruções para entrar na lista, sair da lista e usar a lista em
http://www.mat.puc-rio.br/~nicolau/olimp/obm-l.html
=


[obm-l] Re: [obm-l] Re: [obm-l] Problema de Cálculo

2004-04-16 Por tôpico Henrique Patrício Sant'Anna Branco
Artur,

Agradeço a atenção.
A letra a) eu consegui resolver logo depois que mandei a mensagem (só depois
fui notar que 1/2 e sqrt(3) eram o seno e o cosseno do mesmo angulo).
Mas a letra b) pensava em fazer usando gradientes ou algo assim, pois o
capítulo ao qual pertence o exercício está nessa parte de derivadas
direcionais, gradiente etc.
Alguma idéia?

Em tempo, sou aluno de Estatística e acho sem sentido ensinar Cálculo usando
apenas conceitos físicos (principalmente porque não os conheço direito).

Grato,
Henrique.

 Oi Henrique
 Naum vou poder detalhar uma solucao agora, mas a sua ideia de usar
 coordenadas polares me parece legal. Em vez de velocidade de variacao da
 temperatura (que me parece um termo um tanto infeliz), vamos usar o termo
 taxa de variacao da temparatura. Vc tem que x = r cos(a) e y = r sen(a)
(no
 caso, r=1m), sendo a o angulo que o vetor posicao da particula faz com um
 eixo horizontal de referencia. Substituindo na expressao de T em funcao de
 (x,y), vc obtem T em funcao so de a. Diferenciando com relacao a a, vc
 consegue a taxa de variacao da temperatura com relacao a a. A questao (1)
 pede esta taxa em a = pi/3, e aih eh so substituir.
 Para a questao (2), temos agora que considerar que a depende do tempo t.
Eh
 soh usar a famosa formula do movimento circular uniforme (velocidade
 tangencial = velocidade angular X raio)  para obter a em funcao de t e
 depois usar a regra da cadeia.
 A maior parte do trabalho sao manipulacoes algebricas e uso de formulas
 classicas de diferenciacao.
 Artur

=
Instruções para entrar na lista, sair da lista e usar a lista em
http://www.mat.puc-rio.br/~nicolau/olimp/obm-l.html
=


[obm-l] Re: [obm-l] Re:_[obm-l]_Equação_Trigonométrica!

2004-04-17 Por tôpico Henrique Patrício Sant'Anna Branco
Essa questão é um pouco mais esquisita do que parece.

Vamos fazer i^i = exp(i*ln(i)) e calcular ln(i) por exp(i*t) = cos(t) +
i*sen(t):
exp(i*2k*Pi/2) = i == ln(i) = i*k*Pi (k natural)
Então i^i = exp(i*i*k*Pi) = exp(-k*Pi)

É real, mas é meio estranho pois tem aquela velha história do ln(z) não
estar bem definida para z complexo, com seu valor dependendo do corte que
fizermos no plano complexo.

Henrique.

- Original Message - 
From: [EMAIL PROTECTED]
To: [EMAIL PROTECTED]
Sent: Saturday, April 17, 2004 2:18 AM
Subject: Re: [obm-l] Re:_[obm-l]_Equação_Trigonométrica!


Ja que voce tocou nesta formula...Prove que i^i eh real ! Nao eh complicado.

=
Instruções para entrar na lista, sair da lista e usar a lista em
http://www.mat.puc-rio.br/~nicolau/olimp/obm-l.html
=


Re: [obm-l] funcao e trigonometria

2004-04-19 Por tôpico Henrique Patrício Sant'Anna Branco
Como ninguém respondeu...

A soma da raízes da equação sen^2(x) - sen(x) = 0, para  0 =  x = Pi , é
igual a:

Faça sen^2(x) - sen(x) = sen(x)*(sen(x) - 1) = 0
Agora temos que sen(x) = 0 ou sen(x) - 1 = 0 = sen(x) = 1
Pra x em [0,Pi], temos x = 0, x = Pi e x = Pi/2.

A outra é mais chatinha... Tem que usar a fórmula do vértice da parábola.
a tem que ser positivo e o vértice, maior ou igual a zero nas duas
coordenadas. Tente.

Henrique.


- Original Message - 
From: Guilherme Teles [EMAIL PROTECTED]
To: [EMAIL PROTECTED]
Sent: Sunday, April 18, 2004 11:10 PM
Subject: [obm-l] funcao e trigonometria


Que valores deve apresentar o coeficiente a da função f(x) = ax2 - 2x + 1,
para que ela tenha concavidade voltada para cima e vértice no 1º quadrante?

A soma da raízes da equação sen2 x - sen x = 0, para  0  x , é igual a

alguem sabe essas

=
Instruções para entrar na lista, sair da lista e usar a lista em
http://www.mat.puc-rio.br/~nicolau/olimp/obm-l.html
=


Re: [obm-l] funcao e trigonometria

2004-04-19 Por tôpico Henrique Patrício Sant'Anna Branco
Desculpe, Morgado, não tenho acompanhado fielmente a lista por pura falta de
tempo.
Passo o olho por cima de umas coisas e só...

Henrique.

- Original Message - 
From: Augusto Cesar de Oliveira Morgado [EMAIL PROTECTED]
To: [EMAIL PROTECTED]
Sent: Monday, April 19, 2004 9:12 PM
Subject: Re: [obm-l] funcao e trigonometria


 Eh uma conspiraçao de todos contra mim, para que eu me sinta senil? Eu vi
na
 lista, na semana passada as soluçoes dos dois problemas!
 Morgado

 ==
 Mensagem  enviada  pelo  CIP  WebMAIL  - Nova Geração - v. 2.1
 CentroIn Internet Provider  http://www.centroin.com.br
 Tel: (21) 2542-4849, (21) 2295-3331Fax: (21) 2295-2978
 Empresa 100% Brasileira - Desde 1992 prestando servicos online


 -- Original Message ---
 From: Henrique Patrício Sant'Anna Branco [EMAIL PROTECTED]
 To: [EMAIL PROTECTED]
 Sent: Mon, 19 Apr 2004 17:44:05 -0300
 Subject: Re: [obm-l] funcao e trigonometria

  Como ninguém respondeu...
 
  A soma da raízes da equação sen^2(x) - sen(x) = 0, para  0 =  x =
  Pi , é igual a:
 
  Faça sen^2(x) - sen(x) = sen(x)*(sen(x) - 1) = 0
  Agora temos que sen(x) = 0 ou sen(x) - 1 = 0 = sen(x) = 1
  Pra x em [0,Pi], temos x = 0, x = Pi e x = Pi/2.
 
  A outra é mais chatinha... Tem que usar a fórmula do vértice da
parábola.
  a tem que ser positivo e o vértice, maior ou igual a zero nas duas
  coordenadas. Tente.
 
  Henrique.
 
  - Original Message - 
  From: Guilherme Teles [EMAIL PROTECTED]
  To: [EMAIL PROTECTED]
  Sent: Sunday, April 18, 2004 11:10 PM
  Subject: [obm-l] funcao e trigonometria
 
  Que valores deve apresentar o coeficiente a da função f(x) = ax2 -
  2x + 1, para que ela tenha concavidade voltada para cima e vértice
  no 1º quadrante?
 
  A soma da raízes da equação sen2 x - sen x = 0, para  0  x , é igual a
 
  alguem sabe essas
 
 
=
  Instruções para entrar na lista, sair da lista e usar a lista em
  http://www.mat.puc-rio.br/~nicolau/olimp/obm-l.html
 
=
 --- End of Original Message ---

 =
 Instruções para entrar na lista, sair da lista e usar a lista em
 http://www.mat.puc-rio.br/~nicolau/olimp/obm-l.html
 =


=
Instruções para entrar na lista, sair da lista e usar a lista em
http://www.mat.puc-rio.br/~nicolau/olimp/obm-l.html
=


RE: [obm-l] MAXIMOS E MINIMOS

2004-04-21 Por tôpico Henrique Lima Santana
 Ae pessoal,gostaria de ajudas nas questões sobre maximos e minimos abaixo:

1. decompor o nº N em tres parcelas de sorte q seja maxima a soma dos 
produtos  dessas parcelas, tomadas duas a duas.

2.Investigar qual dos paralelepipedos retangulares de area A tem maior 
volume.

Valeuz

_
MSN Messenger: instale grátis e converse com seus amigos. 
http://messenger.msn.com.br

=
Instruções para entrar na lista, sair da lista e usar a lista em
http://www.mat.puc-rio.br/~nicolau/olimp/obm-l.html
=


[obm-l] Re: [obm-l] Dvida de Conceito

2004-04-21 Por tôpico Henrique Patrcio Sant'Anna Branco
Gustavo,

A funo afim engloba as polinomiais de primeiro grau e as funes
constantes. E uma reta vertical no pode ser grfico de uma funo, pois s
podemos associar um real f(x) a cada x, i.e., cada valor do domnio s pode
ter uma imagem. Acho que isso responde suas perguntas.

Henrique.

- Original Message - 
From: Gustavo [EMAIL PROTECTED]
To: Olmpiada [EMAIL PROTECTED]
Sent: Wednesday, April 21, 2004 3:36 AM
Subject: [obm-l] Dvida de Conceito


A funo afim :
 1) tem como grfico uma qualquer reta,uma reta no vertical  ou uma reta
obrigatriamente inclinada (oblqua )em relao aos eixos?.
2)  sinonimo de funo polinomial do primeiro grau?

3)  uma funo do tipo f(x) = ax+b ,com a e b reais e  a diferente de zero
, ou a pode ser zero ?

  E a funo constante  tipo de funo afim ?

   Fazendo uma breve pesquisa em livros de ensino mdio encontrei respostas
diferentes para cada uma destas perguntas, ou pelo menos deixava dvidas em
relao a estes conceitos , E aqui na lista temos conceno em relao a
essas respostas... gostaria de opnio de vcs. abraos...

=
Instrues para entrar na lista, sair da lista e usar a lista em
http://www.mat.puc-rio.br/~nicolau/olimp/obm-l.html
=


[obm-l] Máximos e mínimos

2004-04-21 Por tôpico Henrique Patrício Sant'Anna Branco
Pessoal,
Mais um problema de Cálculo que estou tendo dificuldades.

Achar os máximos, mínimos e pontos de sela da função
f(x,y) = 1/(x^2+y^2-1)

O único problema que estou encontrando aí são os pontos de fronteira. Sei
que esses pontos são {(x,y) : x^2 + y^2 = 1}, o que dá a cirunferencia de
raio unitário. Mas quais candidatos (pontos) devo tomar?

Grato,
Henrique.

=
Instruções para entrar na lista, sair da lista e usar a lista em
http://www.mat.puc-rio.br/~nicolau/olimp/obm-l.html
=


Re: [obm-l] dúvida

2004-04-21 Por tôpico Eduardo Henrique Leitner
On Wed, Apr 21, 2004 at 09:47:39PM -0300, TSD wrote:
  NO UNIVERSO R, seja S o conjunto solução da inequação 
 -8 = ( x² +16) /x   = 8. Determine o número de elementos de S.

qual eh a sua duvida nesse exercício?

a resposta é 2 

=
Instruções para entrar na lista, sair da lista e usar a lista em
http://www.mat.puc-rio.br/~nicolau/olimp/obm-l.html
=


[obm-l] Re: [obm-l] Máximos e mínimos

2004-04-21 Por tôpico Henrique Patrício Sant'Anna Branco
Na verdade, era pra fazer como você fez mesmo.
A minha dúvida era sobre a necessidade de analisar os pontos de fronteira de
f, para descartar quaisquer outros pontos.
Não é necessário?

Agradeço a ajuda,
Henrique.

- Original Message - 
From: niski [EMAIL PROTECTED]
To: [EMAIL PROTECTED]
Sent: Wednesday, April 21, 2004 9:46 PM
Subject: Re: [obm-l] Máximos e mínimos


 É obrigado utilizar multiplicadores de Lagrange? Vou tomar outro caminho.

 Notacao:
 f[x] derivada parcial de f em relacao x.
 =! diferente de

 Do enunciado temos
 f[x] = -2x/(x^2 + y^2 -1)^2
 f[y] = -2y/(x^2 + y^2 -1)^2

 Os pontos criticos sao achados impondo

 -2x/(x^2 + y^2 -1)^2 = 0 (I)
 e
 -2y/(x^2 + y^2 -1)^2 = 0 (II)

 Temos então de (I)
 impondo x = 0
 Assim de II:
 -2y/(y^2 -1)^2 = 0
 só podemos escolher
 y = 0
 Então um ponto critico é (0,0)
 Para saber se (0,0) corresponde a um minimo local, maximo local ou ponto
 de sela, pode-se utilizar o teste da segunda derivada. Para isso vamos
 avaliar o valor de:
 D = (f[xx](0,0))*(f[yy](0,0)) - [f[xy](0,0)]^2
 D = -4 - 0
 D = -4
 Como D  0 , (0,0) é um ponto de sela.

 Se não for isso me avise para pensarmos mais!

 Henrique Patrício Sant'Anna Branco wrote:

  Pessoal,
  Mais um problema de Cálculo que estou tendo dificuldades.
 
  Achar os máximos, mínimos e pontos de sela da função
  f(x,y) = 1/(x^2+y^2-1)
 
  O único problema que estou encontrando aí são os pontos de fronteira.
Sei
  que esses pontos são {(x,y) : x^2 + y^2 = 1}, o que dá a cirunferencia
de
  raio unitário. Mas quais candidatos (pontos) devo tomar?

 -- 
 Niski - http://www.linux.ime.usp.br/~niski

 [upon losing the use of his right eye]
 Now I will have less distraction
 Leonhard Euler

 =
 Instruções para entrar na lista, sair da lista e usar a lista em
 http://www.mat.puc-rio.br/~nicolau/olimp/obm-l.html
 =


=
Instruções para entrar na lista, sair da lista e usar a lista em
http://www.mat.puc-rio.br/~nicolau/olimp/obm-l.html
=


Re: [obm-l] dúvida

2004-04-22 Por tôpico Eduardo Henrique Leitner
pra começar: x  0

primeira hipótese: x  0

-8x = x^2 + 16   Ex^2 + 16 = 8x

para essa hipótese a única solução é x = -4

segunda hipótese: x  0

-8x = x^2 + 16   Ex^2 + 16 = 8x

para essa hipótese a única solução é x = 4


portanto: S = { -4 ; 4 }

resposta: 2 elementos

On Fri, Apr 23, 2004 at 12:19:02AM -0300, TSD wrote:
 NO UNIVERSO R, seja S o conjunto solução da inequação 
 -8 = ( x² +16) /x   = 8. Determine o número de elementos de S.
 
 como é que eu resolvo esta equação de forma inteligente??

=
Instruções para entrar na lista, sair da lista e usar a lista em
http://www.mat.puc-rio.br/~nicolau/olimp/obm-l.html
=


Re: [obm-l] Fatoração ( IMO )

2004-05-09 Por tôpico Eduardo Henrique Leitner
= (x + y)^7 - (x + y)(x^6 - x^5y + x^4y^2 - x^3y^3 + x^2y^4 - xy^5 + y^6) =
= (x+y)[ (x + y)^6 - (x^6 + y^6 -x^5y - xy^5 + x^4y^2 + x^2y^4 - x^3y^3) ]

aqui tenho uma duvida

o que exatamente significa fatorar? c eh colocar a expressão como sendo o produto de 2 
fatores, essa resposta jah eh valida...

On Sun, May 09, 2004 at 02:32:34PM -0300, Fabio Contreiras wrote:
 Alguem tem ideia de como fatorar isso? Um Abraço!
 
 
 ( x + y )^7 - ( x^7 + y^7 )

=
Instruções para entrar na lista, sair da lista e usar a lista em
http://www.mat.puc-rio.br/~nicolau/olimp/obm-l.html
=


Re: [obm-l] O que eh AO ACASO?

2004-05-07 Por tôpico Eduardo Henrique Leitner
eu chutaria assim: a probabilidade serah a mesma para todas as direções que escolhamos 
a corda, pelo princípio do não tem porque ser diferente, de maneira que a 
probabilidade total serah a mesma que a probabilidade para apenas uma direção de 
cordas.

Tomemos uma direção aleatória então. Tomemos um número muito grande de cordas nesta 
direção igualmente espaçadas por todo o circulo. Uma corda serah de comprimento  
sqrt{3} se traçarmos ela com a distancia no mahximo 1/2 do centro assim, a metade das 
cordas totais que traçamos tem comprimento menor que sqrt{3} e metade tem comprimento 
maior que sqrt{3}, logo, probabilidade = 1/2.
Fazendo o mesmo procedimento aumentando o numero de cordas (sempre mantendo-as 
igalmente espaçadas, pelo principio de que nao ha motivos para que a densidade de 
cordas em uma dada regiao do circulo seja maior que em qualquer outra regiao) 
observamos que o resultado nao muda.

creio que seja isso... gostaria de saber se estou certo em fazer considerações do 
tipo: nao ha motivos para que a densidade de cordas em uma dada regiao do circulo seja 
maior que em qualquer outra regiao. 
É que nunca vih alguém fazendo isso e acho que é meio irreal supor coisas como: a 
metade dos numeros reais existentes entre 1 e 3 estao entre 1 e 2...

abraços,
Eduardo Henrique Leitner

On Fri, May 07, 2004 at 11:57:51PM -0300, Claudio Buffara wrote:
 Esse dah margem pra uma boa discussao:
 
 Escolhe-se, AO ACASO, uma corda de uma circunferencia de raio = 1.
 Qual a probabilidade dessa corda ter comprimento  raiz(3)?
 
 []s,
 Claudio.
 
 =
 Instru??es para entrar na lista, sair da lista e usar a lista em
 http://www.mat.puc-rio.br/~nicolau/olimp/obm-l.html
 =

=
Instruções para entrar na lista, sair da lista e usar a lista em
http://www.mat.puc-rio.br/~nicolau/olimp/obm-l.html
=


[obm-l] Estimação

2004-04-30 Por tôpico Henrique Patrício Sant'Anna Branco
Pessoal, não estou conseguindo fazer esse. É de uma lista de exercício do
meu professor de Método Estatísticos 1.

3. Mostre que nenhuma combinação linear de amostra aleatória simples pode
fornecer melhor estimador que a média amostral \bar{X}.

Grato,
Henrique.

=
Instruções para entrar na lista, sair da lista e usar a lista em
http://www.mat.puc-rio.br/~nicolau/olimp/obm-l.html
=


Re: [obm-l] AJUDA

2004-05-01 Por tôpico Henrique Patrício Sant'Anna Branco
Morgado,

Devo começar a disciplina Inferência Estatística no próximo semestre e já
ouvi falar da falta de boas obras traduzidas sobre o assunto. Não me
incomodo de ler livros em inglês, o problema é consegui-los aqui. O livro
citado é bom? Existem outros bons?

Grato,
Henrique.

- Original Message - 
From: Augusto Cesar de Oliveira Morgado [EMAIL PROTECTED]
To: [EMAIL PROTECTED]
Sent: Saturday, May 01, 2004 8:11 PM
Subject: [obm-l] AJUDA



 Peço adiantadamente desculpas aos colegas pelo off topic, mas tenho um
 problema. Qual o ano em que foi editado o livro Introdução à Inferência
 Estatística, SBM, Heleno Bolfarine e Monica Sandoval?
 Obrigado.
 Morgado


 =
 Instruções para entrar na lista, sair da lista e usar a lista em
 http://www.mat.puc-rio.br/~nicolau/olimp/obm-l.html
 =


=
Instruções para entrar na lista, sair da lista e usar a lista em
http://www.mat.puc-rio.br/~nicolau/olimp/obm-l.html
=


[obm-l] Re: [obm-l] Estimação

2004-05-01 Por tôpico Henrique Patrício Sant'Anna Branco
Morgado,

Não entendi direito. Eu precisaria provar que a condição necessária e
suficiente para o estimador ser não-viciado é que a soma dos coeficientes
seja 1, certo? Como eu procederia com isso?

Grato,
Henrique.

- Original Message - 
From: Augusto Cesar de Oliveira Morgado [EMAIL PROTECTED]
To: [EMAIL PROTECTED]
Sent: Friday, April 30, 2004 8:01 PM
Subject: Re: [obm-l] Estimação


 Se os coeficientes são a(1),...,a(n), o estimador será não-viciado sse
 a(1)+...+a(n)=1 e terá variância mínima sse a(1)^2+...+a(n)^2 for mínimo.
 Multiplicadores de Lagrange ou desigualdades espertas mostrarão que
 a(1)=...=a(n)=1/n.

=
Instruções para entrar na lista, sair da lista e usar a lista em
http://www.mat.puc-rio.br/~nicolau/olimp/obm-l.html
=


Re: [obm-l] Sobre a rpm e Integral

2004-05-03 Por tôpico Henrique Patrício Sant'Anna Branco



Morgado, acho que ele se refere ao fato da Regra de 
Cramer algumas vezes mostrar que um sistema é impossível quando não 
é.
Não consigo lembrar um exemplo disso agora, mas 
existe essa possibilidade.

Alias, obrigado pela ajuda com a demonstração de 
estimação.

Henrique.

  - Original Message - 
  From: 
  Augusto 
  Cesar de Oliveira Morgado 
  To: [EMAIL PROTECTED] 
  Sent: Monday, May 03, 2004 10:44 AM
  Subject: Re: [obm-l] Sobre a rpm e 
  Integral
  O Prof. Elon jamais disse que a técnica é falha, até porque ela 
  não é. O que ele disse é que ela é uma técnica muito ruim para resolver 
  sistemas "grandes", pois exige um número muito grande de operações. == 
  Mensagem enviada pelo CIP WebMAIL - Nova 
  Geração - v. 2.1 CentroIn Internet Provider 
  http://www.centroin.com.br Tel: (21) 2542-4849, (21) 
  2295-3331Fax: (21) 2295-2978 Empresa 100% 
  Brasileira - Desde 1992 prestando servicos online -- 
  Original Message --- From: Alan Pellejero 
  [EMAIL PROTECTED] To: [EMAIL PROTECTED] Sent: Mon, 
  3 May 2004 09:29:49 -0300 (ART) Subject: [obm-l] Sobre a rpm e Integral 
   Pessoal, obrigado pela ajuda!A integral 
  é:/  | x / (senx) dx  /  
E sobre uma prova que estava na rpm, o Prof. Elon comenta que 
  a técnica de Cramer pra determinação de um sistema é falha. Eu fiz a 
  demonstração que estava no livro, olhei atentamente, analisei, mas, mesmo 
  assim, não achei o motivo pelo qual essa técnica é falha. Seria um erro 
  conceitual?Muito obrigado...   
   Ps: sobre as "caudas", eu juro que eu as apago antesnão sei o que 
  acontece...mas de qualquer maneira peço desculpas.
  abraços,  Alan Pellejero   
  
  Yahoo! 
  Messenger - Fale com seus amigos online. Instale 
  agora! --- End of Original Message --- 



Re: [obm-l] somatório

2004-05-16 Por tôpico Eduardo Henrique Leitner
eis uma maneira:


n * 2^0 + (n - 1) * 2^1 + (n - 2)*2^2 + ... + 1*2^(n-1) =
= n[ 2^0 + 2^1 + 2^2 + ... + 2^(n-1) ] - { 1*2^1 + 2*2^2 + 3*2^3 + ... + (n-1)*2^(n-1) 
}  =

partindo do suposto que vc conhece a fórmula da soma dos n primeiros termos de uma PG:

n{ 1[2^n - 1]/[2 - 1]} - {[2^1 + 2^2 + 2^3 + ... + 2^(n-2)] + [2^2 + 2^3 + 2^4 + ... + 
2^(n-1)] + [2^3 + 2^4 + ... + 2^(n-1)] + ... + [2^(n-2) + 2^(n-1)] + [2^(n-1)]}  =

n[2^n - 1] - {2[2^(n-1) - 1] + 2^2[2^(n-2) - 1] + 2^3[2^(n-3) - 1] + ... + 2^(n-2)[2^2 
- 1] + 2^n[2^1 - 1]} =

n[2^n - 1] - [ (2^n - 2) + (2^n - 2^2) + (2^n - 2^3) + ... + [2^n - 2^(n-2)] + [2^n - 
2^(n-1)] =

n2^n - n - [ (n-1)2^n - [ 2 + 2^2 + 2^3 + ... + 2^(n-1) ] =
2^n - n + { 2[2^(n-1) - 1]/[2 - 1] } =
2^n - n + 2^n - 2 =
2^(n+1) - (n+2)

resposta:  2^(n+1) - (n+2)



On Sun, May 16, 2004 at 08:32:39PM -0300, Gustavo Baggio wrote:
 Alguém manja de alguma fórmula pra calcular direto o somatório de
 n * 2^0 + (n - 1) * 2^1 + (n - 2)*2^2 + ... + 1*2^(n-1)  ?
 Isso nada mais é do que somatório de i variando de 0 até (n-1) de 
 (n - i)*(2^i).
 Por exemplo para n = 4 temos 4*1 + 3*2 + 2*4 + 1*8.
  
 Qualquer dica, enfim, tá valendo...
 []'s
  
 Gustavo
 
 
 
 -
 Yahoo! Messenger - Fale com seus amigos online. Instale agora!

=
Instruções para entrar na lista, sair da lista e usar a lista em
http://www.mat.puc-rio.br/~nicolau/olimp/obm-l.html
=


[obm-l] Distância média quadrática

2004-05-17 Por tôpico Henrique Patrício Sant'Anna Branco
Alguém pode me ajudar nesse? É um problema do livro Cálculo, Volume 2, do
George B. Thomas. Tá na parte de integrais múltiplas e o livro não dá
nenhuma dica sobre isso, apenas propõe o problema.

Encontre o valor médio do quadrado da disância do ponto P(x,y) no disco x^2
+ y^2 =1 ao ponto de fronteira A(1,0)

Grato,
Henrique

=
Instruções para entrar na lista, sair da lista e usar a lista em
http://www.mat.puc-rio.br/~nicolau/olimp/obm-l.html
=


[obm-l] Somatório

2004-05-19 Por tôpico Henrique Patrício Sant'Anna Branco
Pessoal,

Alguém sabe resolver isso ou dar alguma indicação? É uma esperança de uma
v.a. geométrica.

Somatório de x*p*(1-p)^x, com x variando entre 0 e infinito.

Grato,
Henrique.

=
Instruções para entrar na lista, sair da lista e usar a lista em
http://www.mat.puc-rio.br/~nicolau/olimp/obm-l.html
=


[obm-l] determinantes

2004-05-24 Por tôpico Eduardo Henrique Leitner
olá, gostaria de saber se existe uma definição exata de determinante de uma matriz... 

é que eu já vi 3 definições distintas e gostaria de saber se todas sao aceitas como 
definições mesmo, ou apenas uma delas é a certa e as outras sao teoremas a partir 
dessa, ou é ainda uma outra além dessa 3...

uma das definições, dada pelo Manoel Paiva, vol 2 é:
O determinante de uma matriz quadrada A = (a_ij)_(nXn), com n = 2, é igual ao 
produto dos elementos da diagonal principal de qualquer matriz triangular B, 
equiparável a A.

bom, nesse caso eu gostaria de saber se existe algum lugar em que eu posso encontra a 
demonstração desses dois teoremas:

Dada uma matriz quadrada A = (a_ij)_(nXn), existe uma matriz triangular B = 
(b_ij)_(nXn) equiparável a A.
esse eu acho meio intuitivo, mas tentei provar matematicamente e não consegui...


Se duas matrizes triangulares A e B são equiparáveis, então ambas possuem o mesmo 
produto dos elementos da diagonal principal.
esse nao é nem um pouco intuitivo e tb nao consegui demonstrar.

bom, a outra definição que encontrei para determinante foi no Gelson Iezzi vol. 4.:
O determinante de uma matriz de ordem n = 2 é a soma dos produtos dos elementos da 
primeira coluna pelos respectivos cofatores.

a outra definição que encontrei foi em um e-mail enviado para esta lista, por Hugo 
Iver Vasconcelos Gonçalves:
o determinante de uma matriz é a soma algébrica de todos os possíveis fatores em que 
estão presentes um (e apenas um) elemento de cada linha e cada coluna,  sendo que 
aqueles em que os índices dos elementos da matriz formam uma permutação de primeira 
classe são tomados positivamente e os demais, negativamente.
nesse caso a explicação que ele deu para permutação de primeira classe foi:
permutação de primeira classe é aquela em que o número de inversões é par
e a explicação para inversões foi:
inversão é o fato de um par de elementos de uma permutação não aparecer na mesma 
ordem que apareceram na permutação inicial.  No caso de a permutação inicial de n 
números ser a disposição deste em ordem crescente, uma inversão seria basicamente o 
fato de aparecer um número maior antes de um menor. E se a ordem inicial deles for 
outra, pode-se sempre chamar o 1o elemento de a1 e o n-ésimo de an, de modo que uma 
inversão será simplesmente quando aparecer um número ap antes de um aq, tais que p  
q.

nesse caso eu nao entendi como calcular quantas inversoes foram necessarias para 
chegar a dada permutação...


bom, é isso, sanadas minha dúvidas e se não for abuso, gostaria de saber onde poderia 
encontrar a demonstração do teorema fundamental de Laplace.

desde já agradeço

=
Instruções para entrar na lista, sair da lista e usar a lista em
http://www.mat.puc-rio.br/~nicolau/olimp/obm-l.html
=


Re: [obm-l] Re:[obm-l] RES: [obm-l] Re:[obm-l] Re:[obm-l] Re: [obm-l] En:livro colégio na val E OUTROS

2004-05-26 Por tôpico Eduardo Henrique Leitner
oras, 30 de dezembro do ano X ela tem 17
  31 de dezembro eh seu aniversario: fez 18
  hoje eh primeiro de janeiro
  esse ano (X+1) ela fará 19 dia 31 de dezembro e ano que vem (X+2) fará 20

On Wed, May 26, 2004 at 11:14:08PM -0300, Osvaldo wrote:
 É mesmo :P , o prob. é esse aqui...
 
 Frida tinha, anteontem, 17 anos e no ano que vem ela 
 fará 20 anos, como isso é possível?
 
 
 
  Não é possível.
  
  -Mensagem original-
  De: [EMAIL PROTECTED] [mailto:owner-obm-
 [EMAIL PROTECTED] Em
  nome de Osvaldo
  Enviada em: Wednesday, May 26, 2004 7:39 PM
  Para: obm-l
  Assunto: [obm-l] Re:[obm-l] Re:[obm-l] Re:[obm-l] 
 En:livro colégio na
  val E OUTROS
  
  
  (Ai Leandro, não quis ofender cara (...) mais enfim
  Minha facu ta em greve, esse brasil, nem pra da um 
  aumento pros prfs.)
  Vo aproveita e deixar um problema, axo ki é clássico 
  não sei Frida tem, anteontem, 17 anos e amanha terá 
  20, como isso é possível? 
  
 Tá ,valeu,você nao foge da razao,nao mandarei mas 
   nada que nao seja matematica
   
   
   
   -- Início da mensagem original ---
   
 De: [EMAIL PROTECTED]
   Para: quot;obm-lquot; [EMAIL PROTECTED]
 Cc: 
   Data: Mon, 24 May 2004 20:43:34 -0300
Assunto: [obm-l]  Re:[obm-l] En:livro colégio 
 naval 
  E 
   OUTROS
   
ESTA LISTA É PARA QUE MESMO? KKK








 -- Início da mensagem original -
 --
 
   De: quot;leandro-epcarquot; leandro-
 [EMAIL PROTECTED]
 Para: quot;obm-lquot; [EMAIL PROTECTED]
   Cc: 
 Data: Thu, 20 May 2004 16:04:51 -0300
  Assunto: livro colégio naval E OUTROS
 
Cheguei numa conclusão que a melhor maneira 
 de 
 passar para frente é enviar uma cópia pelo 
correio.Como 
 a apostila tem 142 páginas mais a encardenação 
 e 
  a 
 postagem vai ficar em torno de R$ 25,00.
Os que se intersarem como nosso amigo 
AURI ,enviem-
 me o endereço por completo e fazendo 
   favor ,deposite 
os 
 R$25,00 na conta abaixo para não me quebrar.;
Para eu identificar o contribuinte ,sejem 
autênticos 
 no deposito,como um deposito de R$ 25,03,de 
 modo 
   que 
eu 
 os identifique.
não esqueção de me avisarem 
 
BANCO: HSBC
  AGÊNCIA: 261
CONTA: 558222
  TITULAR: NOEL JOSE DA COSTA.
 
GRATO COMO SEMPRE
   LEANDRO GERALDO DA COSTA
  
 

   
  
 ___
_
 __
 Acabe com aquelas janelinhas que pulam na sua 
  tela.
 AntiPop-up UOL - É grátis!
 http://antipopup.uol.com.br/
 
 
  
 

   
  
 ___
___
 Acabe com aquelas janelinhas que pulam na sua 
  tela.
 AntiPop-up UOL - É grátis!
 http://antipopup.uol.com.br/
 
 
 
 

   
  
 ===
==
 Instruções para entrar na lista, sair da lista 
 e 
usar a lista em
 http://www.mat.puc-rio.br/~nicolau/olimp/obm-
  l.html
 

   
  
 ===
==
 

Atenciosamente,

Engenharia Elétrica - UNESP Ilha Solteira
Osvaldo Mello Sponquiado 
Usuário de GNU/Linux


 

   
  
 
   __
Acabe com aquelas janelinhas que pulam na sua 
 tela.
AntiPop-up UOL - É grátis!
http://antipopup.uol.com.br/




   
  
 
   =
Instruções para entrar na lista, sair da lista e 
  usar 
   a lista em
http://www.mat.puc-rio.br/~nicolau/olimp/obm-
 l.html

   
  
 
   =


   
  
 
  __
   Acabe com aquelas janelinhas que pulam na sua tela.
   AntiPop-up UOL - É grátis!
   http://antipopup.uol.com.br/
   
   
   
   
  
 
  =
   Instruções para entrar na lista, sair da lista e 
 usar 
  a lista em
   http://www.mat.puc-rio.br/~nicolau/olimp/obm-l.html
   
  
 
  =
   
  
  Atenciosamente,
  
  Engenharia Elétrica - UNESP Ilha Solteira
  Osvaldo Mello Sponquiado 
  Usuário de GNU/Linux
  
  
   
  
 
 
  __
  Acabe com aquelas janelinhas que pulam na sua tela.
  AntiPop-up UOL - É grátis!
  http://antipopup.uol.com.br/
  
  
  
  
 
 
  =
  Instruções para entrar 

Re: [obm-l] ITA 73

2004-05-30 Por tôpico Eduardo Henrique Leitner
tirando o minimo multiplo comum:

(l^2 + m^2 + n^2 + p^2)/lmnp

tratemos do numerador:

l^2 + m^2 + n^2 + p^2 = (l + m)^2 - 2lm + (n + p)^2 - 2np =
= (l + m + n + p)^2 - 2(l+m)(n+p) - 2lm - 2np =
= (l + m + n + p)^2 - 2ln - 2lp - 2mn - 2 mp - 2lm - 2np =
= (l + m + n + p)^2 - 2(ln + ln + lm + nm + np  + mp)

pelas relações de Girard isso eh igual a:

(-q)^2 - 2r = q^2 - 2r

o denominador eh igual a t

logo, 

l/mnp + m/lnp + n/lmp + p/lmn = (q^2 - 2r)/t
acho que estah certo...

On Sun, May 30, 2004 at 08:13:13PM -0300, [EMAIL PROTECTED] wrote:
 oi pessoal! Sou o Rafael Lima aluno do Sistema Elite de Ensino- RJ e queria q vcs me 
 ajuda-se a Fazer essa questão:
 Seja a equação do 4° x^4+qx^3+rx^2+sx+t=0 onde q,r,s,t são números racionais não 
 nulos tais que l,m,n,p são raízes reais dessa equação.
 o valor de l/mnp + m/lnp + n/lmp + p/lmn = ???
 
 
 
 Acesse nosso portal www.click21.com.br
 
 Porque internet grátis, nem a Embratel pode fazer mais barato. Mas pode fazer melhor.
 
 
 =
 Instruções para entrar na lista, sair da lista e usar a lista em
 http://www.mat.puc-rio.br/~nicolau/olimp/obm-l.html
 =

=
Instruções para entrar na lista, sair da lista e usar a lista em
http://www.mat.puc-rio.br/~nicolau/olimp/obm-l.html
=


[obm-l] Re: [obm-l] Re: [obm-l] Continuidade - Exercício

2004-06-08 Por tôpico Henrique Patrício Sant'Anna Branco
Re: [obm-l] Continuidade - ExercícioDepende da questão, mas provar isso é
fácil.

Faça u = exp(x) - 1 e daí, x = ln(1+u)
Ficamos então com lim_x \to 0  u/ln(1+u) = lim_x \to 0  1/ln[(1+u)^(1/u)] =
1/ln(e) = 1, usando só uma propriedade do logaritimo e o limite de
(1+x)^(1/x) com x tendendo a zero, que é igual a e = 2.7182...

Abraço,
Henrique.
- Original Message - 
From: Fellipe Rossi
To: [EMAIL PROTECTED]
Sent: Tuesday, June 08, 2004 10:18 PM
Subject: [obm-l] Re: [obm-l] Continuidade - Exercício


Muito obrigado! Eu tenho prova disso amanha! vc ajudou bastante!! :)

Eu posso dizer que lim(x-0) (e^x - 1)/x = 1 é um limite fundamental?
ou numa prova eu precisaria provar isso?

=
Instruções para entrar na lista, sair da lista e usar a lista em
http://www.mat.puc-rio.br/~nicolau/olimp/obm-l.html
=


[obm-l] O Último Teorema de Fermat

2004-06-09 Por tôpico Henrique Patrício Sant'Anna Branco
Pessoal,

Tenho ouvido muita coisa sobre esse teorema na faculdade e gostaria de
saber, de vocês, se o caso geral já foi demonstrado. Sei que o próprio
Fermat provou sua validade quando 4|n.

Pra quem não sabe do que estou falando, aí vai o enunciado:

A equação diofantina x^n + y^n = z^n não é solúvel por nenhum triplo (x, y,
z), com x, y, z E N, se n  2.

Grato,
Henrique.

=
Instruções para entrar na lista, sair da lista e usar a lista em
http://www.mat.puc-rio.br/~nicolau/olimp/obm-l.html
=


[obm-l] Re: [obm-l] O_Último_Teorema_de_Fermat

2004-06-09 Por tôpico Henrique Patrício Sant'Anna Branco
Valdery,

Valeu. Com o nome do autor, consegui achar o paper onde ele publicou a
prova.
Quem estiver interessado: http://math.stanford.edu/~lekheng/flt/wiles.pdf
Tem 10Mb...

Abraços,
Henrique.

- Original Message - 
From: Valdery Sousa
To: [EMAIL PROTECTED]
Sent: Wednesday, June 09, 2004 4:12 PM
Subject: Re: [obm-l] O_Último_Teorema_de_Fermat


Caro Henrique,
Sobre o teorema referido,saiba que ele já foi comprovado pelo matemático
Andrew Wiles. Apenas não sei onde
vc poderia encontrar essa demonstração disponível.


  Cordialmente,
  Valdery Sousa.

=
Instruções para entrar na lista, sair da lista e usar a lista em
http://www.mat.puc-rio.br/~nicolau/olimp/obm-l.html
=


[obm-l] n^i

2004-06-22 Por tôpico Eduardo Henrique Leitner
alguém pode me dizer qual é o sentido de elevar algum numero à unidade imaginária?

n^i??

se nao ouver sentido, daonde surgiu esses teoremas:

cos (theta) = {e^[i(theta)] + e^[-i(theta)]}/2

sen (theta) = {e^[i(theta)] - e^[-i(theta)]}/(2i)

grato
=
Instruções para entrar na lista, sair da lista e usar a lista em
http://www.mat.puc-rio.br/~nicolau/olimp/obm-l.html
=


[obm-l] Integrais de funções ímpares

2004-06-22 Por tôpico Henrique Patrício Sant'Anna Branco
Pessoal,

Ensinando primitivação (integrais simples) para uma colega, notei que a
integral de funções polinomiais ímpares e da função seno, por exemplo, gera
uma primitiva representada por uma função par.

Isso é um resultado geral? Quero dizer:
Seja f(x) uma função ímpar com primitiva elementar. Int f(x) dx sempre vai
ser uma função par? Como demonstro?

Obrigado.
Henrique.

=
Instruções para entrar na lista, sair da lista e usar a lista em
http://www.mat.puc-rio.br/~nicolau/olimp/obm-l.html
=


[obm-l] Re: [obm-l] Re: O que é limite?

2004-07-14 Por tôpico Henrique Patrício Sant'Anna Branco
Acho que vale a pena dar uma lida nessa página, pra ter uma intuição melhor
sobre derivadas e um esboço de limite.
http://www.pcarv.pro.br/fisica_moderna.htm

Para algo mais preciso (como a definição do epsilon-delta dos limites),
procure um livro de Análise ou parta para Um Curso de Cálculo, vol. I do
Guidorizzi.

Abraço,
Henrique.

- Original Message - 
From: Bruno França dos Reis [EMAIL PROTECTED]
To: [EMAIL PROTECTED]
Sent: Wednesday, July 14, 2004 3:49 PM
Subject: [obm-l] Re: O que é limite?


 -BEGIN PGP SIGNED MESSAGE-
 Hash: SHA1

 On Wednesday 14 July 2004 15:07, Rafael Alves da Silva wrote:
  O QUE É LIMITE DE UMA FUNÇÃO?

 (subject! coloquem subject nas mensagens!)


 Limite de uma função é o valor ao qual ela se aproxima. Por exemplo:
 f(x)=2x+1
 lim(f(x), x-0) = 1
 Veja que nesse caso, o limite tem o mesmo valor da função naquele ponto.
Na
 verdade, isso acontece com qualquer polinômio: lim(x-a, p(x)) = p(a).
 Agora veja outro exemplo:
 g(x)=(sin x)/x
 Claramente vemos que g(x) não está definida para x=0 (pois teriamos uma
 divisão por 0). Mas de qualquer forma, podemos calcular o limite pra isso
ae
 tendendo a 0:
 lim((sin x)/x, x-0) = 1
 Pegue uma calculadora e teste para valores aproximando-se de 0. Quão mais
 próximo de 0 vc colocar o x, mais próximo de 1 ficará (sin x)/x.

 Há outras coisas também, por exemplo:
 f(x) = 1, se x!=0
 f(x) = 2, se x=0
 Temos que:
 lim(f(x),x-0) = 1, mesmo que 1!=f(0).

 Além disso, se temos:
 h(x) = 1, se x0
 h(x) = 2, se x0
 Não existe lim(h(x),x-0), pois existe lim+ e lim-, sendo que o primeiro
seria
 a aproximação vinda pela direita, e o segundo pela esquerda. Como esses
dois
 valores sao diferentes (lim+ = 1, lim- = 2), não há um único limite, entao
 nao há lim, q.e.d.

 Há várias propriedades de limites, mas essas eu vou deixar para vc ver num
 livro (mesmo pq eu posso falar alguma bobagem aqui!)

 Verifique no Fundamentos de Matemática Elementar, vol 8, do Iezzi.
 Muito legal esse livro. Foi o livro com o qual eu comecei a entender
realmente
 o que significava um limite, derivadas e integrais.

 abraço


 - -- 
 Bruno França dos Reis
 brunoreis at terra com br
 icq: 12626000
 gpg-key: http://planeta.terra.com.br/informatica/brunoreis/brunoreis.key

 -BEGIN PGP SIGNATURE-
 Version: GnuPG v1.2.4 (GNU/Linux)

 iD8DBQFA9YBEsHdDIT+qyroRApLiAJ9jA2BAFYEKkTxt08U/RDphJJPnKgCgsU8U
 vGdthKeXfn5w6eASZX5LeMc=
 =dbQn
 -END PGP SIGNATURE-

 =
 Instruções para entrar na lista, sair da lista e usar a lista em
 http://www.mat.puc-rio.br/~nicolau/olimp/obm-l.html
 =


=
Instruções para entrar na lista, sair da lista e usar a lista em
http://www.mat.puc-rio.br/~nicolau/olimp/obm-l.html
=


[obm-l] Re: [obm-l] Máximos e Mínimos

2004-07-15 Por tôpico Henrique Patrício Sant'Anna Branco
Italo,

O que ficou claro pra mim, durante o curso de Cálculo I, foi o fato do teste
da derivada se aplicar apenas a pontos interiores ao intervalo, i.e., que
não estejam em seus extremos. O teorema enuncia Seja f uma função derivável
em p, onde p é um ponto interior ao domínio de f. Uma condição necessária
para que p seja um ponto de máximo ou mínimo local é que f'(p) = 0. Pelo
menos, foi essa a definição que o livro deu a ponto interior em um
intervalo [a,b]: um ponto dentro do intervalo ]a,b[.

Isso parece ser ainda mais verdadeiro quando os livros de Cálculo dão o
roteiro para achar máximo e mínimos:
a) Aplicar o teste da primeira derivada para achar os possíveis pontos
b) Aplicar o teste da segunda derivada para determinar os pontos de inflexão
c) Comparar os valores da função nos pontos obtidos com os valores que esta
toma nos extremos das funções

O terceiro passo parece mostrar, realmente, que f'(p) = 0 certamente não
encontra todos os extremantes.
No caso da função identidade, realmente tal teste não teria nenhum efeito. A
única coisa que poderíamos tirar daí é que a função é estritamente
crescente, pois a derivada primeira é sempre positiva.

Espero que ajude.
Grato,
Henrique.

- Original Message - 
From: [EMAIL PROTECTED]
To: [EMAIL PROTECTED]
Sent: Thursday, July 15, 2004 10:02 AM
Subject: Re: [obm-l] Re: [obm-l] [obm-l] Máximos e Mínimos

Creio que a afirmação seja inversa. Sempre que a derivada for nula então a
função terá um máximo ou um mínimo, ou, ainda, um ponto de inflexão.
Considere, por exemplo, a função f:[a,b]-R,f(x)=x. Temos que ela possui um
máximo e um mínimo em b e a, resp., porém em nenhum dos dois pontos a
derivada se anula.

=
Instruções para entrar na lista, sair da lista e usar a lista em
http://www.mat.puc-rio.br/~nicolau/olimp/obm-l.html
=


[obm-l] Questões estranhas

2004-08-02 Por tôpico Henrique Patrício Sant'Anna Branco
Alguém poderia me dar uma ajuda nisso?

1 - Sabendo-se que a equação x^2*(x + 13) - 6x*(x^2 + 2) + 4 = 0 pode ser
escrita como o produto de dois binômios do primeiro grau, a soma de duas das
suas raízes distintas é igual a:
Resp.: 3

2 - O valor numérico da expressão 120x^4 + 10k^2 + 8, sendo k um natural, é
o quadrado de um número natural para:
Resp.: Nenhum valor de k

Esse eu assumi que a equação pudesse ser fatorada como ((x - a)(x - b))^2 ou
(x - c)^4 para ser um quadrado perfeito, resolvi a biquadrada e aí se chega
à conclusão que não existe nenhuma raiz natural (nem mesmo real) dessa
equação. É o modo certo de fazer?

Grato,
Henrique.

=
Instruções para entrar na lista, sair da lista e usar a lista em
http://www.mat.puc-rio.br/~nicolau/olimp/obm-l.html
=


[obm-l] Outra

2004-08-02 Por tôpico Henrique Patrício Sant'Anna Branco
Um número natural N deixa resta 2 quando dividido por 3, resto 3 quando
dividido por 7 e resto 19 quando dividido por 41. Qual o resto da divisão do
número k = (N + 1)(N + 4)(N + 22) por 861?

=
Instruções para entrar na lista, sair da lista e usar a lista em
http://www.mat.puc-rio.br/~nicolau/olimp/obm-l.html
=


Re: [obm-l] Um de geometria do Claudio Buffara

2004-10-10 Por tôpico Eduardo Henrique Leitner
nao entendih essa parte:

Daí UMVL seria um paralelogramo!

por que seria um paralelogramo?


eu resolvi esse por tangentes...

no triangulo ABC de mediana AD, traçamos a altura em relação ao vértice C (corta a 
reta AB em H) e a altura do trangulo ABD em relação ao vértice D (corta a reta AB em I)

BHC e BID sao semlhantes pois possuem todos os angulos iguais. Como BC = 2BD, podemos 
dizer que BH = 2BI, e vamos chamar a medida BI de x, HC de h, AB de y

finalmente, podemos dizer que tg (IAD) = h/[2(y - x)], tg (BAC) = h/(y - 2x), tg (ABC) 
= h/2x

com essas equações podemos achar uma relação entre as tres tangentes que nao depende 
nem de h, nem de x e nem de y, portanto, o angulo ABC estah determinado unicamente 
pelos angulos IAD e BAC, que sao iguais para o triangulo PQR e portanto, o angulo PQR 
eh igual ao angulo ABC e portanto os triangulos sao semelhantes...

(é, eu também acho que a minha solução deu bem mais trabalho... hehehe)
e nessa resolução eu nao considerei o caso de os angulos em questao serem retos, mas 
se o forem eh muito fahcil provar que eles sao semelhantes...



On Sun, Oct 10, 2004 at 06:20:35PM -0300, Tércio Miranda wrote:
 Problema
 São dados os triângulos ABC e PQR, com medianas AD e PS , respectivamente. Valem as 
 seguintes igualdades de
 Ângulos, BAD=QPS e CAD=RPS. Prove que ABC e PQR são semelhantes.
 
 Fixemos o triângulo ABC no seu plano.
 Consideremos as semiretas AB e AC. Sobre elas marquemos os pontos L e M tal que 
 AL=PQ e AM=QR. As hipóteses nos
 dão as congruências dos triângulos PQR e ALM (LAL).
 A reta suporte da mediana AD corta o segmento de reta LM num ponto K, o qual pelas 
 hipóteses de igualdade de ângulos
 BAD=QPS e CAD=RPS, é o ponto médio do segmento LM.
 Agora, se, por absurdo, LM não for paralela a BC podemos conduzir por K uma paralela 
 a BC que cortará AB e AC (semiretas)
 nos pontos U e V, respectivamente. Daí UMVL seria um paralelogramo! Um contradição.
 Então LM é paralela a BC e os triângulos ABC e ALM são semelhantes e temos o 
 resultado.
 
 Um abraço do colega 
 Tércio Miranda
   
=
Instruções para entrar na lista, sair da lista e usar a lista em
http://www.mat.puc-rio.br/~nicolau/olimp/obm-l.html
=


Re: [obm-l] Um de geometria do Claudio Buffara

2004-10-11 Por tôpico Eduardo Henrique Leitner
aa é verdade

valeu!

On Mon, Oct 11, 2004 at 11:10:55AM -0300, Tércio Miranda wrote:
 Porque as diagonais do referido quadrilátero intersectar-se-iam pelo ponto
 médio, daí um paralelogramo.
 Certo?
 Saludos
 Tércio Miranda
 - Original Message -
 From: Eduardo Henrique Leitner [EMAIL PROTECTED]
 To: [EMAIL PROTECTED]
 Sent: Sunday, October 10, 2004 9:35 PM
 Subject: Re: [obm-l] Um de geometria do Claudio Buffara
 
 
  nao entendih essa parte:
 
  Daí UMVL seria um paralelogramo!
 
  por que seria um paralelogramo?
 
 
  eu resolvi esse por tangentes...
 
  no triangulo ABC de mediana AD, traçamos a altura em relação ao vértice C
 (corta a reta AB em H) e a altura do trangulo ABD em relação ao vértice D
 (corta a reta AB em I)
 
  BHC e BID sao semlhantes pois possuem todos os angulos iguais. Como BC =
 2BD, podemos dizer que BH = 2BI, e vamos chamar a medida BI de x, HC de h,
 AB de y
 
  finalmente, podemos dizer que tg (IAD) = h/[2(y - x)], tg (BAC) = h/(y -
 2x), tg (ABC) = h/2x
 
  com essas equações podemos achar uma relação entre as tres tangentes que
 nao depende nem de h, nem de x e nem de y, portanto, o angulo ABC estah
 determinado unicamente pelos angulos IAD e BAC, que sao iguais para o
 triangulo PQR e portanto, o angulo PQR eh igual ao angulo ABC e portanto os
 triangulos sao semelhantes...
 
  (é, eu também acho que a minha solução deu bem mais trabalho... hehehe)
  e nessa resolução eu nao considerei o caso de os angulos em questao serem
 retos, mas se o forem eh muito fahcil provar que eles sao semelhantes...
 
 
 
  On Sun, Oct 10, 2004 at 06:20:35PM -0300, Tércio Miranda wrote:
   Problema
   São dados os triângulos ABC e PQR, com medianas AD e PS ,
 respectivamente. Valem as seguintes igualdades de
   Ângulos, BAD=QPS e CAD=RPS. Prove que ABC e PQR são semelhantes.
  
   Fixemos o triângulo ABC no seu plano.
   Consideremos as semiretas AB e AC. Sobre elas marquemos os pontos L e M
 tal que AL=PQ e AM=QR. As hipóteses nos
   dão as congruências dos triângulos PQR e ALM (LAL).
   A reta suporte da mediana AD corta o segmento de reta LM num ponto K, o
 qual pelas hipóteses de igualdade de ângulos
   BAD=QPS e CAD=RPS, é o ponto médio do segmento LM.
   Agora, se, por absurdo, LM não for paralela a BC podemos conduzir por K
 uma paralela a BC que cortará AB e AC (semiretas)
   nos pontos U e V, respectivamente. Daí UMVL seria um paralelogramo! Um
 contradição.
   Então LM é paralela a BC e os triângulos ABC e ALM são semelhantes e
 temos o resultado.
  
   Um abraço do colega
   Tércio Miranda
  
  =
  Instruções para entrar na lista, sair da lista e usar a lista em
  http://www.mat.puc-rio.br/~nicolau/olimp/obm-l.html
  =
 
 
 =
 Instruções para entrar na lista, sair da lista e usar a lista em
 http://www.mat.puc-rio.br/~nicolau/olimp/obm-l.html
 =
=
Instruções para entrar na lista, sair da lista e usar a lista em
http://www.mat.puc-rio.br/~nicolau/olimp/obm-l.html
=


Re: [obm-l] duvida sobre permutas e análise combinatória

2004-10-11 Por tôpico Eduardo Henrique Leitner
C(10, 5) = 10!/[(5!)(5!)] = 10*9*8*7*6/(5*4*3*2) = 9*8*7*6/(4*3)= 9*8*7*6/(2*6) = 
9*8*7/2 = 9*4*7 = 36*7 = 252

C(2, 1) = 2!/[(1!)(1!)] = 2

C(10, 5)*C(2, 1) = 252 * 2 = 504

=)

On Mon, Oct 11, 2004 at 07:12:59PM -0300, Lucy Santos wrote:
 Leo,
 obrigada, mas o x da questão é que C(10,5)*C(2,1) dá 240 (multiplicando por 2).Por 
 favor, me mostre um passo a passo de sua conta, pois devo estar errando no início.
 Grata
 lucy
 Credo estou até com vergonha de repostar esta dúvida, mas não consegui mesmo 
 resolver sozinha!
 
 Leonardo Paulo Maia [EMAIL PROTECTED] wrote:
 Olá a todos, acabo de aderir à lista. Lucy, o seu raciocínio está no caminho
 certo. O problema é que o número que você calculou, C(10,5) * C(2,1) = 504,
 representa a quantidade de grupos de 6 pessoas onde há um paulista e 5
 não-paulistas. O problema é que, quando você seleciona 6 pessoas para compor um
 grupo, você automaticamente gerou um segundo grupo, o dos excluídos. Se a
 pergunta fosse de quantas formas posso constituir UM grupo a partir de 12
 pessoas, onde ..., a resposta 504 estaria correta. Mas a pergunta é de
 quantas formas as 12 pessoas podem ser divididas: C(10,5) * C(2,1) conta duas
 vezes o número de divisões possíveis do grupo inicial. Ok? Até mais, Leo.
 
 Quoting Lucy Santos :
 
  Pessoal,
  sei que minha pergunta é bem humilde em relação aos temas normalmente
  propostos aqui neste grupo, mas realmente gostaria de resolver esta questão e
  preciso da ajuda de vocês.
  Por favor, já tentei resolver e cheguei à c(10,5) *C(2,1) e deu 240, sei que
  a resposta correta é a d, mas para isso não sei como deu 504..
  Agradeço se poderem me enviar um passo a passo.
  1- Um grupo formado por 12 pessoas tem 2 paulistas.de quantas formas pode
  ocorrer esta divisão para 2 grupos de 6 pessoas de forma em que cada grupo
  haja um paulista?
  a)180 b)200 c)226 d)252 e)300
  
  
  
  
  
  -
  Yahoo! Acesso Grátis - Internet rápida e grátis. Instale o discador agora!
 
 
 =
 Instruções para entrar na lista, sair da lista e usar a lista em
 http://www.mat.puc-rio.br/~nicolau/olimp/obm-l.html
 =
 
   
 -
 Yahoo! Acesso Grátis - Internet rápida e grátis. Instale o discador agora!
=
Instruções para entrar na lista, sair da lista e usar a lista em
http://www.mat.puc-rio.br/~nicolau/olimp/obm-l.html
=


Re: [obm-l] Problema Legal

2004-10-15 Por tôpico Eduardo Henrique Leitner
o 96 tb seria azul nao?
seguindo sua lohgica:

3
6
12
24
48
96, cujo maior divisor impar eh 3: 4*0 + 3

entao teremos 48 azuis e portando 52 vermelhos, correto?


On Wed, Oct 13, 2004 at 10:19:59AM -0200, Claudio Buffara wrote:
 on 12.10.04 18:09, benedito at [EMAIL PROTECTED] wrote:
 
  Abaixo, segue um problema legal:
  
  
  
  Problema
  
  Num corredor, existem  100  armários  em fila, numeradas de  1  até  100. Um
  pintor vem e pinta todas os armários de vermelho. Em seguida, vem um segundo
  pintor e pinta de azul os armários de três em três, começando do armário
  número 3. A seguir, vem um terceiro pintor e pinta de vermelho os armários
  de cinco em cinco, começando no armário de número 5 (ele pinta de vermelho,
  mesmo que o armário já seja vermelho). Em seguida, vem um quarto pintor e
  pinta de azul os armários de sete em sete, começando no armário  7. A
  seguir, vem um quinto pintor, e assim por diante, alternando a pintura
  vermelha, azul, até o pintor de número 50.
  
  No final, quantos armários são vermelhos?
 
  
 As pinturas vermelhas sao multiplas de 1, 5, 9, 13, ..., 97 (4k+1)
 As pinturas azuis sao multiplas de 3, 7, 11, 15, ..., 99 (4k+3)
 
 A cor final de um dado armario eh a cor da ultima pintura que ele recebe e
 esta cor corresponde justamente ao maior divisor impar do numero do armario.
 Assim, o numero de armarios azuis eh igual ao numero de inteiros entre 1 e
 100 (inclusive) cujo maior divisor impar eh da forma 4k+3.
 
 Estes numeros sao:
 3, 7, 11, 15, ..., 95, 99   (25);
 6, 14, 22, 30, ..., 86, 94   (25+12=37);
 12, 28, 44, 60, 76, 92   (37+6=43);
 24, 56, 88   (43+3=46);
 48   (46+1=47)
 
 Logo, os armarios vermelhos sao em numero de 100 - 47 = 53.
 
 []s,
 Claudio.
 
 
 
 =
 Instruções para entrar na lista, sair da lista e usar a lista em
 http://www.mat.puc-rio.br/~nicolau/olimp/obm-l.html
 =
=
Instruções para entrar na lista, sair da lista e usar a lista em
http://www.mat.puc-rio.br/~nicolau/olimp/obm-l.html
=


Re: [obm-l] mais de probabilidade

2004-10-15 Por tôpico Eduardo Henrique Leitner
o das probabilidades eh moh braçal (pelo menos nao encontrei nenhuma maneira mais 
inteligente de fazê-la)

probabilidade de levar exatamente um tiro*0,05 + prob de levar exatamente 2 tiros*0,2 
+ prob de levar exatamente 3 tiros)*4

(0,2*0,6*0,9 + 0,4*0,8*0,9 + 0,1*0,8*0,6)*0,05 + (0,2*0,4*0,9 + 0,2*0,6*01 + 
0,8*0,4*0,1)*0,2 + 0,2*0,4*0,1*0,4

resolvendo esses calculos eu cheguei em 0,0486 = 4,86%


On Wed, Oct 13, 2004 at 05:28:01AM -0200, Murilo Neves wrote:
 Olá
 
 Continuo aqui estudando e surgiram mais duas dúvidas. Se alguém puder me 
 ajudar... Vamos lá:
 
 1)Sabendo-se que E(X)=2 e que E(X^2)=4, calcule o menor valor possível para 
 P(-10X14).
 Meu gabarito dá que a resposta é aproximadamente 0.9. Tentei usar 
 Chebyshev, mas como Var(X)=E^2(X) - E(X^2)=0, chegaria a resposta 1. Tentei 
 usar Markov, mas também não consegui.
 
 2) Três cruzadores A, B e C, atacam um navio inimigo. Na primeira leva de 
 tiros, o cruzador A tem 0.2 de probabilidade de atingir o alvo; enquanto os 
 cruzadores B e C têm, respectivamente, 0.4 e 0.1 de probabilidade. Se o 
 navio inimigo receber tiros de apenas um dos navios, ele tem prob 0.05 de 
 afundar imediatamente; se receber de dois, a prob aumenta para 0.2 e se 
 receber de três, a prob é de 0.4. Qual a prob. do navio inimigo afundar na 
 primeira leva de tiros?
 
 Obrigado
 
 _
 MSN Hotmail, o maior webmail do Brasil.  http://www.hotmail.com
 
 =
 Instruções para entrar na lista, sair da lista e usar a lista em
 http://www.mat.puc-rio.br/~nicolau/olimp/obm-l.html
 =
=
Instruções para entrar na lista, sair da lista e usar a lista em
http://www.mat.puc-rio.br/~nicolau/olimp/obm-l.html
=


Re: [obm-l] Geom. Plana

2004-10-15 Por tôpico Eduardo Henrique Leitner
trapezóide eh o mesmo que trapézio?

On Wed, Oct 13, 2004 at 10:39:25AM -0300, aryqueirozq wrote:
 01. Um círculo é inscrito em um trapezóide ABCD.Tome 
 K,L,M,N como os pontos de intersecço~es do círculo com 
 as diagonais AC e BD respectivamente ( K entre A e L ,  
 e M entre B e N ) . Sendo AK*LC = 16 e BM * ND = 9/4 , 
 ache o raio do círculo.
 
 
 
 02.Suponha que n( r ) denota o número de pontos com 
 coordenadas inteiras em um círculo de raio r  1. Prove 
 que n ( r )  6*raiz cúbica de pi*r^2 
   Agradeço.
  
 __
 Acabe com aquelas janelinhas que pulam na sua tela.
 AntiPop-up UOL - É grátis!
 http://antipopup.uol.com.br/
 
 
 
 =
 Instruções para entrar na lista, sair da lista e usar a lista em
 http://www.mat.puc-rio.br/~nicolau/olimp/obm-l.html
 =
=
Instruções para entrar na lista, sair da lista e usar a lista em
http://www.mat.puc-rio.br/~nicolau/olimp/obm-l.html
=


Re: [obm-l] Provar uma congruencia

2004-10-16 Por tôpico Eduardo Henrique Leitner
acho que a minha resposta tah bacana:

F(n) = n^5 - 20n^4 + 40n^3 + 70n^2 + 79n - 50

para reduzir o grau dessa expressao, podemos utilizar uma outra que sabemos que eh 
multipla de 120:
por exemplo: (n-5)(n-4)(n-3)(n-2)(n-1)
esse numero eh multiplo de 120 pois eh multiplo de 5 (produto de 5 numeros 
consecutivos), eh multiplo de 3 (produto de tres numeros consecutivos) e eh multiplo 
de 8 (produto de pelo menos dois numeros pares consecutivos)

(n-5)(n-4)(n-3)(n-2)(n-1) = n^5 - 15n^4 + 85n^3 - 225n^2 + 274n - 120

e portanto: F(n) = (n-5)(n-4)(n-3)(n-2)(n-1) - 5n^4 - 45n^3 + 295n^2 - 195n + 70
F(n) = (n-5)(n-4)(n-3)(n-2)(n-1) - 5( n^4 + 9n^3 - 59n^2 + 39n - 14)

entao, para provar que F(n) = 0 (mod 120), para qualquer primo maior que 7, basta 
provar que P(n) = n^4 + 9n^3 - 59n^2 + 39n - 14 = 0 (mod 24), para qualquer primo 
maior que 7.

agora vamos utilizar o mesmo procedimento: sabemos que o produto de quatro numeros 
consecutivos eh multplo de 24, vamos usar a expressao n(n+1)(n+2)(n+3) = n^4 + 6n^3 + 
11n^2 + 6n

portanto, P(n) = n(n+1)(n+2)(n+3) + 3n^3 - 70n^2 + 33n - 14

entao, para provar que F(n) = 0 (mod 120) para qualquer primo maior que 7, basta 
provar que Q(n) = 3n^3 - 70n^2 + 33n - 14 = 0 (mod 24) para qualquer primo maior que 7.

se o numero é primo e maior que 7, entao ele é ímpar e pode ser escrito como n = 2k+1

Q(2k+1) = 3[2k + 1]^3 - 70[2k + 1]^2 + 33[2k + 1] - 14 = 4[6k^3 - 61k^2 - 49k - 12]

portanto, basta provar que 6k^3 - 61k^2 - 49k - 12 é multiplo de 6

multiplo de 2 percebos que eh, pois se k for impar teremos:

par - impar - impar - par = par

se k for par, teremos:

par - par - par - par = par

entao, soh falta provar que é multiplo de 3, o que será se 61k^2 + 49k o for.

caso k = 3p, 61k^2 + 49k = k(61k + 49) = 3p(61k + 49), que eh multiplo de 3
k = 3p + 1 faria com que o numero n fosse igual a 3(3p + 1), o que contraria a 
hipótese de que n é primo e maior que 7
caso k = 3p + 2,  61k^2 + 49k = 3[183p^2 + 293p + 114], que eh multiplo de 3


como soh falatava provar isso, a tese estah provada 

On Wed, Oct 13, 2004 at 07:19:28PM -0300, Demetrio Freitas wrote:
 Ola,
 
 Gostaria de provar uma congruencia. 
 
 Dado F(n) = n^5 -20*n^4 +40*n^3 +70*n^2 +79*n -50 
 Prove que F(n) = 0 (mod 120), se n for primo  7.
 (Onde = denota conguente)
 
 Por exemplo:
 F(11) = -69240 = -120 * 577
 F(19) = 170760 =  120 * 1423
 F(97) = 6853927800 = 120 * 57116065
 F(563) = 54562015773960 = 120 * 454683464783
 
 Porem:
 F(15) = -101240 - nao divisivel por 120
 F(129) = 30271636600 - nao divisivel por 120
 F(597) = 73303331579800 - nao divisivel por 120
 
 
 Qual caminho usar?
 
 Obrigado,
 
 Demetrio
 
 OBS:
 Naturalmente a condição eh se n primo e não sse (se
 e somente se), pois ha muitos n compostos onde F(n) 
 = 0 (mod 120)
 
 
 
   
   
   
 ___ 
 Yahoo! Acesso Grátis - Internet rápida e grátis. Instale o discador agora! 
 http://br.acesso.yahoo.com/
 =
 Instruções para entrar na lista, sair da lista e usar a lista em
 http://www.mat.puc-rio.br/~nicolau/olimp/obm-l.html
 =
=
Instruções para entrar na lista, sair da lista e usar a lista em
http://www.mat.puc-rio.br/~nicolau/olimp/obm-l.html
=


Re: [obm-l] 1000 primeiros dígitos de n^1998

2004-10-16 Por tôpico Eduardo Henrique Leitner
o primeiro digito é o das unidades ou o de maior valor?

On Sat, Oct 16, 2004 at 10:45:59PM -0400, [EMAIL PROTECTED] wrote:
 Olá pessoal !
 
 Prove que existe n pertencente a N tal que os 1000 primeiros dígitos de 
 n^1998 são iguais a 1.
 
 
 
=
Instruções para entrar na lista, sair da lista e usar a lista em
http://www.mat.puc-rio.br/~nicolau/olimp/obm-l.html
=


Re: [obm-l] Probabilidade

2004-10-21 Por tôpico Eduardo Henrique Leitner
considerando 53 cartas (13 de cada naipe e um curinga) e nao considerando que o ás 
seja um numero.

probabilidade de sair dois numeros, pelo menos um nao de copas:

(27/53)(35/52) + (9/53)(27/52) = 297/689

probabilidade de sair duas cartas de copas:

(13/53)(12/52) = 3/53

total:

297/689 + 3/53 = 336/689

acho que estah certo, se nao estiver, por favor corrijam


On Thu, Oct 21, 2004 at 08:34:52PM -0300, Daniela Yoshikawa wrote:
 De um baralho tira-se duas cartas ao acaso. Qual a probabilidade de sair dois 
 números ou duas cartas de copas?
 
   
 -
 Yahoo! Acesso Grátis - Internet rápida e grátis. Instale o discador agora!
=
Instruções para entrar na lista, sair da lista e usar a lista em
http://www.mat.puc-rio.br/~nicolau/olimp/obm-l.html
=


Re: [obm-l] Problema sobre complexos [ime 2003-2004]

2004-10-22 Por tôpico Eduardo Henrique Leitner
olha, eu nao sei o que o ime pretendia com essa questao, mas achei uma solução bem 
interessante sem pensar muito...:

z = -1

a = 1; b = 2; c = 3

hehehe, acho que nao tem solução mais simples que essa...

On Thu, Oct 21, 2004 at 07:21:42PM -0700, Felipe Torres wrote:
 Oi.
 eu resolvi o problema a seguir e gostaria de saber se
 a resposta est? correta, j? que n?o h? uma ?nica
 solu??o.
 
 Sendo a, b e c n?meros naturais em progress?o
 aritm?tica e z um n?mero complexo de m?dulo unit?rio,
 determine um valor para cada um dos n?meros a, b, c e
 z de forma que eles satisfa?am a igualdade:
 
 1/z^a + 1/z^b + 1/z^c = z^9
 
 eu achei como uma solu??o poss?vel
 z= cis(pi/2)
 a=-8
 b=-9
 c=-10
 
 
   
 ___
 Do you Yahoo!?
 Declare Yourself - Register online to vote today!
 http://vote.yahoo.com
 =
 Instru??es para entrar na lista, sair da lista e usar a lista em
 http://www.mat.puc-rio.br/~nicolau/olimp/obm-l.html
 =
=
Instruções para entrar na lista, sair da lista e usar a lista em
http://www.mat.puc-rio.br/~nicolau/olimp/obm-l.html
=


Re: [obm-l] Cachorrada (probabilidade)

2004-10-22 Por tôpico Eduardo Henrique Leitner
1o: 50%, sao eventos independentes
2o: 50% + 25% = 75%  (admitindo a probabilidade do segundo cao ser macho ou femea ser 
igual)
3o: 

olha, eu tentei, tentei, parei, pensei e cheguei a seguinte conclusao:

é impossivel, soh podem ser expressos por tais relações os polinomios que sao 
simetricos

nesse caso, por exemplo, a relação de a com c nao se da da mesma maneira que a de b 
com c, ou a de a com b, entao é impossivel

sagita: de ser a conjugação na terceira pessoa do presente do indicativo do verbo 
sagitar, que significa dar forma de seta a

On Fri, Oct 22, 2004 at 02:51:23PM -0300, Jozias Del Rios (ToniK) wrote:
 Tenho dois cachorros. Pelo menos um deles é macho. Qual a 
 probabilidade dos dois serem machos? (ok...)
 Se agora escolhermos um dos cachorros ao acaso, qual a 
 probabilidade dele ser macho?
 
 Algum de vocês ja tentou escrever o polinomio ba^2 + cb^2 + 
 ac^2 em função das funções simétricas elementares (a+b+c), 
 (ab+bc+ca), (abc) ?
 
 O que eh sagita ??
 
 []'s
  
 __
 Acabe com aquelas janelinhas que pulam na sua tela.
 AntiPop-up UOL - É grátis!
 http://antipopup.uol.com.br/
 
 
 
 =
 Instruções para entrar na lista, sair da lista e usar a lista em
 http://www.mat.puc-rio.br/~nicolau/olimp/obm-l.html
 =
=
Instruções para entrar na lista, sair da lista e usar a lista em
http://www.mat.puc-rio.br/~nicolau/olimp/obm-l.html
=


Re: [obm-l] Re:[obm-l] Um problema de Física ( Densidade)

2004-10-22 Por tôpico Eduardo Henrique Leitner
observe que a sua resposta eh impossivel, pois se a densidade da gasolina fosse maior 
que da agua, e o volume de liquido em ambas as pesagens for o mesmo, o balde com 
gasolina deveria pesar mais, e nao eh o que ocorre...

On Fri, Oct 22, 2004 at 04:30:21PM -0300, Osvaldo Mello Sponquiado wrote:
 Olá !, minha tentativa é:
 
 
 [Adotando g=9,81 m/s^2; p_agua=10^3 kg/m^3]
 
 Frasco: m_frasco.9,81=0,12=m_frasco=12,23g
 Água: P_água=0,52-0,12=0,4N=(10^3.V).9,81=0,4=V=4.10^-5 m^3
 
 Supondo que o volume ocupado pela água e a gasolina seja o MESMO, temos:
 
 P_gasolina=0,42-0,12=0,3N= (p_gas.4.10^-5).9.81=0.3=7,645 . 10^3 kg/m^3, ou seja,
 7,6 vezes mais pesado do que a água.
 
 []'s
 
 
  
  Colegas, me ajudem a tirar um dúvida neste problema:
  
  Um frasco de vidro pesa 0,12N. Este frasco, cheio de gasolina, pesa 0,42N 
  e, cheio d'água pesa 0,52N. Determine a densidade absoluta dessa amostra de 
  gasolina, em kg/m^3.
  
  Depois de fazerem uns cálculos estranhos  me garantiram que a resposta é  
  0,18 x 10^4 kg/m^3.
  E o pior é que era essa a resposta na apostila.
  
  Me ajudem, pois minha resposta é outra.
  
  (^_^)
  
  _
  MSN Messenger: converse com os seus amigos online.  
  http://messenger.msn.com.br
  
  =
  Instruções para entrar na lista, sair da lista e usar a lista em
  http://www.mat.puc-rio.br/~nicolau/olimp/obm-l.html
  =
  
 
 Atenciosamente,
 
 Osvaldo Mello Sponquiado 
 Engenharia Elétrica, 2ºano 
 UNESP - Ilha Solteira
 
  
 __
 Acabe com aquelas janelinhas que pulam na sua tela.
 AntiPop-up UOL - É grátis!
 http://antipopup.uol.com.br/
 
 
 
 =
 Instruções para entrar na lista, sair da lista e usar a lista em
 http://www.mat.puc-rio.br/~nicolau/olimp/obm-l.html
 =
=
Instruções para entrar na lista, sair da lista e usar a lista em
http://www.mat.puc-rio.br/~nicolau/olimp/obm-l.html
=


Re: [obm-l] eq. de terceiro grau

2004-10-22 Por tôpico Eduardo Henrique Leitner
isso me faz lembrar que tenho uma duvida a respeito da fórmula de cardano
utilizando ela, como obtenho as tres raizes? tipo, utilizo raízes analogas em cada 
raiz cubica?

raizes análogas: utilizando a fórmula de Moivre pra calcular as raizes cubicas eu 
coloco k=0 na primeira e k=0 na segunda, obtendo uma das raizes; depois coloco k=1 e 
ambas e acho a segunda raiz e depois k=2 em ambas e acho a 3a raiz

porque essa foi a unica maneira que consegui pensar que me retornaria exatamente 3 
raizes...

agradeço respostas


On Fri, Oct 22, 2004 at 07:49:24PM +, Edward Elric wrote:
 Como isso eh muito chato de digitar aqui olhe esse site 
 http://www.teorema.mat.br/phpBB2/viewtopic.php?t=54
 ae tem a soluçao da equaçao de terceiro grau generica.
 
 From: eritotutor [EMAIL PROTECTED]
 Reply-To: [EMAIL PROTECTED]
 To: obm-l [EMAIL PROTECTED]
 Subject: [obm-l] eq. de terceiro grau
 Date: Fri, 22 Oct 2004 16:32:56 -0300
 
 Num problema do curso de farmacia  apareceu a seguinte equação:
 
 an^3 + nb +1 = 0 , onde a,b são maiores de zero.
 
 []s
 
 __
 Acabe com aquelas janelinhas que pulam na sua tela.
 AntiPop-up UOL - É grátis!
 http://antipopup.uol.com.br/
 
 
 _
 MSN Messenger: converse com os seus amigos online.  
 http://messenger.msn.com.br
 
 =
 Instruções para entrar na lista, sair da lista e usar a lista em
 http://www.mat.puc-rio.br/~nicolau/olimp/obm-l.html
 =
=
Instruções para entrar na lista, sair da lista e usar a lista em
http://www.mat.puc-rio.br/~nicolau/olimp/obm-l.html
=


Re: [obm-l] eq. de terceiro grau

2004-10-22 Por tôpico Eduardo Henrique Leitner
a unica maneira que eu conheço é dividindo todos os termos por a e aplicando a fórmula 
de Cardano...

isso me faz lembrar que tenho uma duvida a respeito da fórmula de cardano
utilizando ela, como obtenho as tres raizes? tipo, utilizo raízes analogas e cada raiz 
cubica?

raizes análogas: utilizando a fórmula de Moivre pra calcular as raizes cubicas eu 
coloco k=0 na primeira e k=0 na segunda, obtendo uma das raizes; depois coloco k=1 e 
ambas e acho a segunda raiz e depois k=2 em ambas e acho a 3a raiz

porque essa foi a unica maneira que consegui pensar que me retornaria exatamente 3 
raizes...

agradeço respostas

On Fri, Oct 22, 2004 at 04:32:56PM -0300, eritotutor wrote:
 Num problema do curso de farmacia  apareceu a seguinte equação:
 
 an^3 + nb +1 = 0 , onde a,b são maiores de zero.
 
 []s
  
 __
 Acabe com aquelas janelinhas que pulam na sua tela.
 AntiPop-up UOL - É grátis!
 http://antipopup.uol.com.br/
 
=
Instruções para entrar na lista, sair da lista e usar a lista em
http://www.mat.puc-rio.br/~nicolau/olimp/obm-l.html
=


Re: [obm-l] eq. de terceiro grau

2004-10-22 Por tôpico Eduardo Henrique Leitner
mm, eh uma possibilidade, mas porque a formula de cardano nos retorna apenas uma das 
raizes? nao me lembro de nenhuma restrição feita na demonstração...

On Fri, Oct 22, 2004 at 09:25:51PM +, Edward Elric wrote:
 Depois de achar a primeira raiz por Cardano use Briot-Ruffini que vai cair 
 num polinomio de segundo grau ae eh facil.
 
 From: Eduardo Henrique Leitner [EMAIL PROTECTED]
 Reply-To: [EMAIL PROTECTED]
 To: [EMAIL PROTECTED]
 Subject: Re: [obm-l] eq. de terceiro grau
 Date: Fri, 22 Oct 2004 19:19:31 -0200
 
 a unica maneira que eu conheço é dividindo todos os termos por a e 
 aplicando a fórmula de Cardano...
 
 isso me faz lembrar que tenho uma duvida a respeito da fórmula de cardano
 utilizando ela, como obtenho as tres raizes? tipo, utilizo raízes analogas 
 e cada raiz cubica?
 
 raizes análogas: utilizando a fórmula de Moivre pra calcular as raizes 
 cubicas eu coloco k=0 na primeira e k=0 na segunda, obtendo uma das 
 raizes; depois coloco k=1 e ambas e acho a segunda raiz e depois k=2 em 
 ambas e acho a 3a raiz
 
 porque essa foi a unica maneira que consegui pensar que me retornaria 
 exatamente 3 raizes...
 
 agradeço respostas
 
 On Fri, Oct 22, 2004 at 04:32:56PM -0300, eritotutor wrote:
  Num problema do curso de farmacia  apareceu a seguinte equação:
 
  an^3 + nb +1 = 0 , onde a,b são maiores de zero.
 
  []s
 
  
 __
  Acabe com aquelas janelinhas que pulam na sua tela.
  AntiPop-up UOL - É grátis!
  http://antipopup.uol.com.br/
 
 =
 Instruções para entrar na lista, sair da lista e usar a lista em
 http://www.mat.puc-rio.br/~nicolau/olimp/obm-l.html
 =
 
 _
 MSN Messenger: converse com os seus amigos online.  
 http://messenger.msn.com.br
 
 =
 Instruções para entrar na lista, sair da lista e usar a lista em
 http://www.mat.puc-rio.br/~nicolau/olimp/obm-l.html
 =
=
Instruções para entrar na lista, sair da lista e usar a lista em
http://www.mat.puc-rio.br/~nicolau/olimp/obm-l.html
=


Re: [obm-l] JOGO DE BARGANHA!

2004-10-22 Por tôpico Eduardo Henrique Leitner
por acaso a resposta eh: o A fica em melhor situação, recebendo $90 e o B $10 ?

vamos lidar com os naturais e supor que nenhum jogador tenha raiva do outro, por 
exemplo:

na visao do B, entre ele nao receber nada e o A ganhar 50, ou ambos nao receberem 
nada, o B vai escolher que A ganhe 50 (isso eh soh pra simplificar, senao teriamos que 
lidar com numeros do tipo $79, mas nao vai mudar muito a solução a essencia eh a mesma)

eu analisei o problema do final para o começo:

supondo que o B recusou a primeira e o A recusou a segunda.
nesse caso, a oferta mais sabia de A serah: 80 pra mim e nada pra vc, certo?

portanto, se o A recusar a segunda, B nao recebe nada, entao B nao deixará em hipótese 
alguma que A recuse a segunda.

entao B recusou a primeira, e ainda nao fez a oferta:

para que A nao recuse a segunda, B deve assegurar que o lucro de A seja maior que 80, 
entao, a oferta mais sabia que B poderia fazer eh: 80 para você e 10 pra mim, ok?

entao, de qualquer maneira, o lucro maximo de A, caso B recuse a primeira serah 80

A consegue ganhar mais do que oitenta se B nao recusar a primeira, e B nao recusará a 
primeira se o lucro de B na primeira oferta for maior ou igual a 10, pois, como vimos, 
esse eh o mahximo que receberá se recusar a primeira.

entao, a primeira oferta mais sabia que A tem a fazer eh: 90 pra mim e 10 pra vc ok?
e B responderah: fazer o que neh...

certo?

On Wed, Oct 20, 2004 at 09:16:45PM -0300, [EMAIL PROTECTED] wrote:
 Turma! Eis um convite aos simpatizantes da indigesta Teoria dos Jogos.
 Divirtam-se!
 
 O jogador A executa o primeiro movimento, fazendo uma oferta ao jogador B para a
 divisão de $100. (Por exemplo, o jogador A poderia sugerir que ele ficasse com
 $60 e o jogador B levasse $40); O jogador B pode aceitar ou recusar a oferta.
 Se ele rejeitar, o montante de dinheiro disponível cairá para $90; O jogador B,
 então, fará uma oferta para a divisão do dinheiro; Se o jogador A rejeitar essa
 oferta, o montante de dinheiro cairá para $80, seguindo-se uma nova oferta do
 jogador A para a divisão; Se o jogador B rejeitar, o montante de dinheiro cairá
 para $0. Ambos os jogadores são racionais, totalmente informados e querem
 maximizar seus lucros. Qual jogador se sairá melhor nesse jogo?
 
 A propósito, um consumidor que começou como emprestador passou a ser tomador
 após o declínio da taxa de juros. Ficou em melhor ou pior situação?
Abraços!
 
 
 __
 WebMail UNIFOR - http://www.unifor.br.
 =
 Instruções para entrar na lista, sair da lista e usar a lista em
 http://www.mat.puc-rio.br/~nicolau/olimp/obm-l.html
 =
=
Instruções para entrar na lista, sair da lista e usar a lista em
http://www.mat.puc-rio.br/~nicolau/olimp/obm-l.html
=


Re: [obm-l] ainda sobre media

2004-10-23 Por tôpico Eduardo Henrique Leitner
acho que nesse caso usaria-se média aritmética ponderada mesmo...

serah  [ 2(1800/2) + 1(1800/1) + 3(1800/3)]/6 = 1800/2 = 900

o problema classico de média harmônica em física é:

um movel percorre a metade de um caminho a uma velocidade v1 e a outra metade a uma 
velocidade v2, qual eh a velocidade mehdia no percurso?

a resposta serah dada pela mehdia harmonica das duas velocidades...

On Fri, Oct 22, 2004 at 10:09:50PM -0200, Gustavo wrote:
 Será que podemos usar média harmônica neste quesito? justifique!!
 
 # diariamente uma rádio divide diariamente um prêmio de $1.800,00,igualmente entre 
 seus vencedores.Nos ultimos três dias tivemos 2,1 e 3 vencedores.Qual foi o prêmio 
 médio? # comentem sobre este peoblema. agradeço a atenção
=
Instruções para entrar na lista, sair da lista e usar a lista em
http://www.mat.puc-rio.br/~nicolau/olimp/obm-l.html
=


Re: [obm-l] IME

2004-10-26 Por tôpico Eduardo Henrique Leitner

eu acho que essa questao estava errada. o que eles queriam que a gnt provasse devia 
ser justamente que

c^2 = (ac)^log[a](b), foi um erro de impressão...

na minha prova eu simplesmente coloquei que o teorema só era válido para os casos em 
que b = d. Acho que eles terao que considerar questao integral pra todos que chegaram 
na conclusão que vc postou ou equivalentes...

On Tue, Oct 26, 2004 at 09:16:06PM -0200, Ariel de Silvio wrote:
 Olá a todos,
 
 Começaram hoje as provas do IME. Hoje foi realizada a prova de matemática.
 Lembro que ano passado propuseram na lista resoluções das questões
 diferentes da resoluções dadas pelos cursinhos. Esse ano vão fazer também?
 
 O Poliedro (www.sistemapoliedro.com.br) está resolvendo. O GPI diz que irá
 resolver também (www.gpi.g12.br). O Poliedro está colocando o enunciado em
 apenas algumas das questões.
 
 Mas já começo com um pedido, a questão 3. Vou passar direto aqui.
 
 Sejam a, b, c, d números reais positivos e diferentes de 1. Sabendo que
 log[a](d), log[b](d) e log[c](d) são termos consecutivos de um progressão
 aritmética, demonstre que:
 c^2 = (ac)^log[a](d)
 
 log[a](d) é log de d na base a
 
 Só que ninguém que conversei conseguiu chegar nisso. Apenas em:
 
 c^2 = (ac)^log[a](b)
 
 Cheguei nisso, e não vejo motivo para b = d
 
 De resto tiveram questões MUITO simples, outras malvadas e outras realmente
 difíceis.
 A questão 4 por exemplo dava duas equações de quarto grau, pedia as raizes
 comuns. Porém não tinha raízes comuns! Cruel pra quem tá ali fazendo a prova
 .
 
 []s
 Ariel
 
 =
 Instruções para entrar na lista, sair da lista e usar a lista em
 http://www.mat.puc-rio.br/~nicolau/olimp/obm-l.html
 =
=
Instruções para entrar na lista, sair da lista e usar a lista em
http://www.mat.puc-rio.br/~nicolau/olimp/obm-l.html
=


Re: [obm-l] IME

2004-10-26 Por tôpico Eduardo Henrique Leitner
essa nao era difícil nao... basta fazer uma pesquisa de raízes racionais na primeira 
equação. vc descobre que 3 e -3 sao raízes

ou vc enxerga a fatoração:

x^4 - 2x^3 - 11x^2 + 18x + 18 = (x^2 - 9)(x^2 - 2x - 2)

nao eh uma fatoração complicada, mas se vc enxergá-la de sozinho, garanto que vc se 
enquadra em um grupo muito seleto de pessoas nesse mundo! eu nao enxergaria

bom, feito isso vc descobre as outras raízes facilmente: 1 +- sqrt{3}

daih eh soh substituir esses quatro valores na segunda equação (na verdade o 3 e o -3 
vc nem precisa substituir, pois 52 nao eh multiplo de 3)

e como tem aquele teorema que diz que se a + sqrt{b} eh raiz, sendo sqrt{b} um 
numero irracional, entao a - sqrt{b} também o é, vc soh precisa substituir uma das 
raízes irracionais...

daih vc ve que nao dá e acabou a questão, nao há raízes comuns.

uma coisa que eu percebi que muitos colegas meus fizeram foi subtrair uma equação da 
outra, e chegar em uma equação de 3o grau, que parecia ser mais simples... hehehe, 
eles nao chegaram a nada com isso... eh um caminho bem traiçoeiro pois a pessoa pensa: 
eu nao vou começar do 0 pois jah consegui abaixar o grau pra 3! soh preciso achar 1 
raiz e tah resolvido o meu problema!
e daí o tempo paaassa, e paassa... hehehe, realmente nao era uma boa idéia...


On Tue, Oct 26, 2004 at 09:40:01PM -0200, Bernardo wrote:
 Essa questão está errada. Só pode estar.
 
 Uma coisa muito estranha nela é que o que ela pede pra vc provar nem tem o 
 b.
 Eu provei na prova que estava errado.
 E isso é fogo pq eu perdi muito tempo tentando chegar no que o enunciado 
 pedia.
 
 Como se fazer a 4ª questão??
 Enunciado:
 Determine o valor das raízes comuns das equações
 x^4 - 2x^3 - 11x^2 + 18x + 18 = 0, e
 x^4 -12x^3 -44x^2 -32x -52 = 0
 
 Abraços
 Bernardo
 
 - Original Message - 
 From: Ariel de Silvio [EMAIL PROTECTED]
 To: [EMAIL PROTECTED]
 Sent: Tuesday, October 26, 2004 9:16 PM
 Subject: [obm-l] IME
 
 
 Olá a todos,
 
 Começaram hoje as provas do IME. Hoje foi realizada a prova de matemática.
 Lembro que ano passado propuseram na lista resoluções das questões
 diferentes da resoluções dadas pelos cursinhos. Esse ano vão fazer também?
 
 O Poliedro (www.sistemapoliedro.com.br) está resolvendo. O GPI diz que irá
 resolver também (www.gpi.g12.br). O Poliedro está colocando o enunciado em
 apenas algumas das questões.
 
 Mas já começo com um pedido, a questão 3. Vou passar direto aqui.
 
 Sejam a, b, c, d números reais positivos e diferentes de 1. Sabendo que
 log[a](d), log[b](d) e log[c](d) são termos consecutivos de um progressão
 aritmética, demonstre que:
 c^2 = (ac)^log[a](d)
 
 log[a](d) é log de d na base a
 
 Só que ninguém que conversei conseguiu chegar nisso. Apenas em:
 
 c^2 = (ac)^log[a](b)
 
 Cheguei nisso, e não vejo motivo para b = d
 
 De resto tiveram questões MUITO simples, outras malvadas e outras 
 realmente
 difíceis.
 A questão 4 por exemplo dava duas equações de quarto grau, pedia as raizes
 comuns. Porém não tinha raízes comuns! Cruel pra quem tá ali fazendo a 
 prova
 .
 
 []s
 Ariel
 
 =
 Instruções para entrar na lista, sair da lista e usar a lista em
 http://www.mat.puc-rio.br/~nicolau/olimp/obm-l.html
 =
 
 
 
 
 =
 Instruções para entrar na lista, sair da lista e usar a lista em
 http://www.mat.puc-rio.br/~nicolau/olimp/obm-l.html
 =
=
Instruções para entrar na lista, sair da lista e usar a lista em
http://www.mat.puc-rio.br/~nicolau/olimp/obm-l.html
=


Re: [obm-l] UM POUCO DE FÍSICA!

2004-10-27 Por tôpico Eduardo Henrique Leitner
sobre a água, se vc nao estiver lidando na temperatura de 0 a 4 graus celsius, a 
densidade da agua diminui com o aumento da temperatura.

assim, para volumes iguais, vc terah menos massa no recipiente de agua quente. e, 
quanto menor a massa, menor será o peso

On Wed, Oct 27, 2004 at 09:34:48PM -0300, [EMAIL PROTECTED] wrote:
 Uma engrenagem de 24 dentes, solta, gira em volta de uma engrenagem com 360
 dentes, fixa. Quando a engrenagem pequena voltar a sua posição inicial, quantas
 voltas ela terá dado em torno de si mesma?
 
 Em uma balança viciada, ao inverter as posições de uma mesma quantidade de
 mercadoria para o segundo freguês eu terei lucro ou prejuízo?
 
 Com relação ao problema dos pêndulos de pesos e tamanhos diferentes abandonados
 de uma certa altura, é o comprimento do cordão que faz com que o pêndulo vá
 mais depressa ou devagar. O peso não tem qualquer influência.
 
 A propósito, existe alguma diferença de peso entre 1 litro de água quente e 1
 litro de água fria?
 
 Grato pela atenção!
 
 
 
 __
 WebMail UNIFOR - http://www.unifor.br.
 =
 Instruções para entrar na lista, sair da lista e usar a lista em
 http://www.mat.puc-rio.br/~nicolau/olimp/obm-l.html
 =
=
Instruções para entrar na lista, sair da lista e usar a lista em
http://www.mat.puc-rio.br/~nicolau/olimp/obm-l.html
=


Re: [obm-l] IME

2004-10-29 Por tôpico Eduardo Henrique Leitner
equivalente grama nao eh obsoleto coisa nenhuma! só porque alguém resolveu tirar o 
conteúdo do programa de ensino médio nao quer dizer que seja obsoleto!

o conceito de equivalente grama é muito útil e foi baseado em equivalentes grama que 
Faraday enunciou as leis da estequiometria na eletroquímica

se você olhar no manual do candidato do IME está escrito lah: 

QUÍMICA

11. Reações e equações químicas: Tipos de reações químicas. Ajuste das equações 
químicas. Número de oxidação. Conceito de oxidação e redução. EQUIVALENTE-GRAMA. 
Estequiometria.

portanto, uma vez que está no programa e foi cobrado em vestibulares recentes do 
ime... nao ha porque reclamar

mas esse problema do numero de avogadro jah tah muito manjado mesmo... hehehe

On Sun, Oct 29, 2000 at 09:42:10PM -0200, Diego Cardoso wrote:
 Sei q jah eh fugir d+ da matematica, mas ainda no IME desse ano, acho
 simplesmente ridiculo se pedir um conceito tão antigo e já obsoleto como o
 Equivalente Grama numa prova de Quimica. E ainda pior é pedir pra demonstrar
 N Avogadro por uma maneira tão decorada de livros...decepcionante...
 Tomara q a banca seja mais criativa e moderna no proximo ano
 
 - Original Message -
 From: Felipe Torres [EMAIL PROTECTED]
 To: [EMAIL PROTECTED]
 Sent: Wednesday, October 27, 2004 10:20 AM
 Subject: Re: [obm-l] IME questão do logaritmo
 
 
  oi
  Só p botar mais um desenvolvimento q mostra q o
  enunciado tava claramente errado:
 
  c^2 = (ac)^loga(d)
 
  c^2/c^loga(d)= d
 
  c^(2-loga(d))=c^(logc(d))
  igualando os expoentes
 
  2 - loga(d) = logc(d)
 
  log a(d)+ log c(d) = 2
 
  aí  deveria ter um logb(d) do lado do dois ali
 
  logo b deveria ser igual a d para questão estar
  correta
 
  {}s
  Felipe
 
 
  __
  Do You Yahoo!?
  Tired of spam?  Yahoo! Mail has the best spam protection around
  http://mail.yahoo.com
  =
  Instruções para entrar na lista, sair da lista e usar a lista em
  http://www.mat.puc-rio.br/~nicolau/olimp/obm-l.html
  =
 
 
 
 
 =
 Instruções para entrar na lista, sair da lista e usar a lista em
 http://www.mat.puc-rio.br/~nicolau/olimp/obm-l.html
 =
=
Instruções para entrar na lista, sair da lista e usar a lista em
http://www.mat.puc-rio.br/~nicolau/olimp/obm-l.html
=


Re: [obm-l] Re: [obm-l] RESOLUÇÕES ENGENHOSAS!

2004-11-06 Por tôpico Eduardo Henrique Leitner
no problema 1, eu soh nao entendih como sao feitas essas chaves... para que o 
campeonato funcione dessa maneira, nao deveria ter um numero de participantes igual a 
uma potencia de 2?

On Fri, Oct 15, 2004 at 11:03:07PM -0300, Paulo Rodrigues wrote:
 1) Em cada partida existe um perdedor e cada jogador, com exceção do
 campeão, perde exatamente uma vez. Logo o total de partidas é igual 341.
 
 2) Cada dominó cobre uma casa preta e uma branca. Retirando duas casas em
 cantos opostos estaremos tirando 2 de mesma cor. Sobrarão 30 de uma cor e
 32 da outra cor. Como cada dominó cobre2 casas de cores diferentes, será
 impossível fazer a cobretura porque 30 é diferente de 32.
 - Original Message -
 From: [EMAIL PROTECTED]
 To: [EMAIL PROTECTED]
 Sent: Friday, October 15, 2004 8:35 PM
 Subject: [obm-l] RESOLUÇÕES ENGENHOSAS!
 
 
 Oi, pessoal!  vamos tentar descobrir os atalhos dos problemas abaixos!
 Divirtam-se!
 
 Um torneio de tênis tem 342 jogadores. Uma única partida envolve dois
 jogadores.
 O vencedor de uma partida vai jogar com o vencedor de uma outra partida na
 próxima rodada, enquanto os perdedores são eliminados do torneio. Os 2
 jogadores que venceram todas as partidas nas rodadas anteriores vão jogar no
 final e o vencedor ganha o torneio. Prove que o número total de partidas que
 serão jogadas é 341.
 
 Um tabuleiro de damas ou de xadrez padrão consiste em 8 fileiras de 8
 quadrados
 cada. Quadrados adjacentes têm cores alternadas, branco e preto (ou vermelho
 e
 preto). Um conjunto de 32 ladrilhos 1x2, cada um cobrindo 2 quadrados,
 cobrem o
 tabuleiro completamente (4 ladrilhos por fileira, 8 fileiras). Prove que, se
 os
 quadrados nos cantos diagonalmente opostos do tabuleiro forem removidos, o
 que
 resta do tabuleiro não pode ser coberto com 31 ladrilhos.
 Abraços!
 
 
 
 __
 WebMail UNIFOR - http://www.unifor.br.
 =
 Instruções para entrar na lista, sair da lista e usar a lista em
 http://www.mat.puc-rio.br/~nicolau/olimp/obm-l.html
 =
 
 
 ---
 Outgoing mail is certified Virus Free.
 Checked by AVG anti-virus system (http://www.grisoft.com).
 Version: 6.0.776 / Virus Database: 523 - Release Date: 12/10/2004
 
 =
 Instruções para entrar na lista, sair da lista e usar a lista em
 http://www.mat.puc-rio.br/~nicolau/olimp/obm-l.html
 =
=
Instruções para entrar na lista, sair da lista e usar a lista em
http://www.mat.puc-rio.br/~nicolau/olimp/obm-l.html
=


Re: [obm-l] Limite e continuidade

2004-11-06 Por tôpico Eduardo Henrique Leitner
bom, eu nao entendo muito de limites, mas esse parece simples

como x  0, se n - inf, entao x*n^2 - inf

fazendo y = x*n^2, temos que lim[y-inf] f(y) = a

On Sat, Nov 06, 2004 at 09:53:12PM -0200, Fabio Niski wrote:
 Pessoal, por favor, quem souber poderia por favor resolver esse:
 
 Suponha f : (0,+inf) - R é uma funcao continua tal que
 lim[n-+inf] f(x*n^2) = a para todo x. (n é inteiro). Prove que 
 lim[x-+inf] f(x) = a
 
 obrigado.
 
 Niski
 
 
 =
 Instruções para entrar na lista, sair da lista e usar a lista em
 http://www.mat.puc-rio.br/~nicolau/olimp/obm-l.html
 =
=
Instruções para entrar na lista, sair da lista e usar a lista em
http://www.mat.puc-rio.br/~nicolau/olimp/obm-l.html
=


Re: [obm-l] EXERCICIO

2004-11-06 Por tôpico Eduardo Henrique Leitner

acho que conseguih... fatore assim:

X1 + X2(1 - X1 - X3) + X3 + X4(1 - X3 - X5) + X5 + X6(1 - X5 -X7) + X7 + ... + X98(1 - 
X97 - x99) + X97 + X100(1 - X99 - X1) + X99

considerando a verdade absoluta: pelo menos metade dos termos é menor ou igual a 0,5 
ou pelo menos metade dos termos é maior ou igual a 0,5

caso ocorra a primeira hipotese, façamos com que os termos de ordem impar sejam 
menores ou iguais a 0,5 (a ordem nao faz diferença mesmo...)

para que a soma seja mahxima, os termos de ordem par devem ser mahximos, ou seja, 
iguais a 1, pois soh aparecem multiplicados por numeros positivos

assim, a soma da 50

caso ocorra a segunda hipótese, façamos com que os termos de ordem impar sejam maiores 
ou iguais a 0,5

para que a soma seja mahxima, os termos de ordem par devem ser minimos, ou seja, 
iguais a 0, pois soh aprecem multiplicados por numeros negativos

dessa forma, a soma se resume à soma dos termos de ordem impar, que serah mahxima 
quando estes forem iguais a um, resultando em 50


portanto, a soma máxima é 50, demonstradamente

On Sat, Nov 06, 2004 at 08:44:56PM -0200, vinicius wrote:
 oi alguem resolve???
 
 
 QUAL O VALOR MÁXIMO PARA:
 X1(1-X2)+X2(1-X3)+...+X100(1-X1)
 ONDE X VARIA DE 0 A 1 E X1 É O PRIMEIRO TERMO, X2 O SEGUNDO
 QUERIA Q DEMONSTRASSE, POIS SEI UM JEITO DE FAZER, MAS SEM DEM.
=
Instruções para entrar na lista, sair da lista e usar a lista em
http://www.mat.puc-rio.br/~nicolau/olimp/obm-l.html
=


Re: [obm-l] EXERCICIO

2004-11-06 Por tôpico Eduardo Henrique Leitner
pensando bem, nao posso fazer isso:

 caso ocorra a primeira hipotese, façamos com que os termos de ordem impar sejam 
 menores ou iguais
a 0,5 (a ordem nao faz diferença mesmo...)

e isso:

 caso ocorra a segunda hipótese, façamos com que os termos de ordem impar sejam 
 maiores ou iguais a 0,5

pois a ordem faz diferença sim.

voltemos a essa forma fatorada:

X1 + X2(1 - X1 - X3) + X3 + X4(1 - X3 - X5) + X5 + X6(1 - X5 - X7) + X7 + ... + X98(1 
- X97 - x99) + X99 + X100(1 - X99 - X1) 

se por acaso a soma de dois termos de ordem impar consecutivos for maior que 1, 
faremos o termo de ordem par correspondente = 0, pois maximizaria aquela parcela 
(tornando-a iguais a 0, em vez de um numero negativo). se por acaso a soma de dois 
termos de ordem impar consecutivos for menor que 1, faremos o termo de ordem par 
correspondente = 1.

os termos de ordem par assumem apenas valores iguais a 0 ou iguais a 1

de maneira analoga, podemos fatorar assim:

X1(1 - X100 - X2) + X2 + X3(1 - X2 - X4) + X4 + x5(1 - X4 - X6) + X6 + ... + X99(1 - 
X98 - X100) + X100

dessa maneira, concluimos que os termos de ordem impar tb assumem apenas valores 
iguais a 0 ou iguais a 1.

voltemos a essa forma:

X1 + X2(1 - X1 - X3) + X3 + X4(1 - X3 - X5) + X5 + X6(1 - X5 - X7) + X7 + ... + X98(1 
- X97 - x99) + X99 + X100(1 - X99 - X1) 

se todos os termos forem iguais a 0, a soma serah igual a 0, entao pelo menos um termo 
deve ser igual a 1

jamais conseguiremos obter na soma essa sequencia:

1 + 1

o 1 sempre virá seguido de um zero, entao nao eh interessante, em hipotese alguma, 
deixar dois zeros seguidos, pois nao poderemos compensá-los depois

portanto, a soma mahxima será:

1 + 0 + 1 + 0 + 1 + 0 +... + 1 + 0 = 50

portanto, a soma máxima é 50

agora acho que tá certinho, mas acho que me enrolei demais...

On Sun, Nov 07, 2004 at 12:50:32AM -0200, Eduardo Henrique Leitner wrote:
 
 acho que conseguih... fatore assim:
 
 X1 + X2(1 - X1 - X3) + X3 + X4(1 - X3 - X5) + X5 + X6(1 - X5 -X7) + X7 + ... + X98(1 
 - X97 - x99) + X97 + X100(1 - X99 - X1) + X99
 
 considerando a verdade absoluta: pelo menos metade dos termos é menor ou igual a 0,5 
 ou pelo menos metade dos termos é maior ou igual a 0,5
 
 caso ocorra a primeira hipotese, façamos com que os termos de ordem impar sejam 
 menores ou iguais a 0,5 (a ordem nao faz diferença mesmo...)
 
 para que a soma seja mahxima, os termos de ordem par devem ser mahximos, ou seja, 
 iguais a 1, pois soh aparecem multiplicados por numeros positivos
 
 assim, a soma da 50
 
 caso ocorra a segunda hipótese, façamos com que os termos de ordem impar sejam 
 maiores ou iguais a 0,5
 
 para que a soma seja mahxima, os termos de ordem par devem ser minimos, ou seja, 
 iguais a 0, pois soh aprecem multiplicados por numeros negativos
 
 dessa forma, a soma se resume à soma dos termos de ordem impar, que serah mahxima 
 quando estes forem iguais a um, resultando em 50
 
 
 portanto, a soma máxima é 50, demonstradamente
 
 On Sat, Nov 06, 2004 at 08:44:56PM -0200, vinicius wrote:
  oi alguem resolve???
  
  
  QUAL O VALOR MÁXIMO PARA:
  X1(1-X2)+X2(1-X3)+...+X100(1-X1)
  ONDE X VARIA DE 0 A 1 E X1 É O PRIMEIRO TERMO, X2 O SEGUNDO
  QUERIA Q DEMONSTRASSE, POIS SEI UM JEITO DE FAZER, MAS SEM DEM.
 =
 Instruções para entrar na lista, sair da lista e usar a lista em
 http://www.mat.puc-rio.br/~nicolau/olimp/obm-l.html
 =
=
Instruções para entrar na lista, sair da lista e usar a lista em
http://www.mat.puc-rio.br/~nicolau/olimp/obm-l.html
=


[obm-l] maximo

2004-11-11 Por tôpico Eduardo Henrique Leitner
olá pessoal, eu não consigo de jeito nenhum achar o máximo dessa expressão:


n/{5 + [1/(a_1)] + [1/(a_2)] + [1/(a_3)] + ... + [1/(a_n)]}

em que todas as letras (n, a_1, a_2, a_3, ..., a_n) pertencem ao naturais nao nulos e:

a_1 + a_2 + a_3 + ... + a_n = 32

obrigado por qualquer ajuda
=
Instruções para entrar na lista, sair da lista e usar a lista em
http://www.mat.puc-rio.br/~nicolau/olimp/obm-l.html
=


Re: [obm-l] maximo

2004-11-12 Por tôpico Eduardo Henrique Leitner
genial! muito obrigado Cláudio! com isso deu pra resolver esse problema de física do 
ITA:

Você dispõe de um dispositivo de resistência R = 5r, e 32 baterias idênticas, cada 
qual com resistência r e força eletromotriz V. Como seriam associadas as baterias, de 
modo a obter a máxima corrente que atravesse R? Justifique.


mas caramba, colocarem isso numa prova de 30 questoes pra 4 horas é desumano!

será que havia alguma maneira mais simples de chegar a essa resposta, sem ter que 
achar esse máximo que pedi?

On Fri, Nov 12, 2004 at 09:00:14AM -0200, Claudio Buffara wrote:
 on 11.11.04 23:08, Eduardo Henrique Leitner at [EMAIL PROTECTED]
 wrote:
 
  olá pessoal, eu não consigo de jeito nenhum achar o máximo dessa expressão:
  
  
  n/{5 + [1/(a_1)] + [1/(a_2)] + [1/(a_3)] + ... + [1/(a_n)]}
  
  em que todas as letras (n, a_1, a_2, a_3, ..., a_n) pertencem ao naturais nao
  nulos e:
  
  a_1 + a_2 + a_3 + ... + a_n = 32
  
 
 Para n fixo, o valor maximo da expressao ocorre quando:
 1/a_1 + ... + 1/a_n eh minimo.
 
 Como os a_i sao inteiros positivos, o valor minimo de 1/a_1 + ... + 1/a_n
 vai ocorrer quando eles forem o mais proximo possivel uns dos outros.
 Basta ver que para x positivo e |x|  |a|  |b|, vale sempre:
 1/(x+a) + 1/(x-a) = 1/(x+b) + 1/(x-b), com igualdade == |a| = |b|.
 
 Ou seja, supondo que a_1 = a_2 = ... = a_n, teremos que:
 a_1 = ... = a_r = [32/n]+1 e a_(r+1) = ... = a_n = [32/n],
 onde: 
 r = resto da divisao de 32 por n;
 [x] = parte inteira de x = piso(x).
 (repare que isso garante que a_1 + ... + a_n = 32)
 
 Assim, dado n, o valor maximo da expressao serah igual a:
 f(n) = n/(5 + r/([32/n]+1) + (n-r)/[32/n]).
 
 Naturalmente, n deve ser = 32. Caso contrario, a restricao dos a_i serem
 inteiros nao seria obedecida.
 
 Em suma, basta calcular o valor de f(n) para n = 1, 2, ..., 32 e tomar o
 maior deles. Com um computador isso eh feito facilmente resultando em:
 n = 11 == [32/n] = 2, r = 10 == f(n) = 11/(5 + 10/3 + 1/2) = 66/53
 
 Ou seja, o valor maximo da expressao eh igual a 66/53 e ocorre quando:
 n = 11,  a_1 = ... = a_10 = 3,  a_11 = 2.
 
 
 []s,
 Claudio.
 
 
 =
 Instruções para entrar na lista, sair da lista e usar a lista em
 http://www.mat.puc-rio.br/~nicolau/olimp/obm-l.html
 =
=
Instruções para entrar na lista, sair da lista e usar a lista em
http://www.mat.puc-rio.br/~nicolau/olimp/obm-l.html
=


Re: [obm-l] Duvidas II

2004-11-13 Por tôpico Eduardo Henrique Leitner
pode-se resolver a inequação 2x+5 / 8 - 3x e obter uma solução S1
resolve-se a inequação 2x+5 = 35 e obtém-se uma solução S2

faz-se a interseção das soluções S1 e S2 e chega-se a uma solução S3

resolve-se a inequação 8 - 3x = 2x + 5 e obtém-se uma solução S4
resolve-se a inequação 8 - 3x = 35 e obtém-se uma solução S5

faz-se a interseçao das soluçoes S4 e S5 e chega-se a uma solução S6

a solução da questao é dada pela uniao da soluçao S3 com a solução S6

essa foi a primeira maneira que pensei, mas a solução que o Artur apresentou é mais 
rápida e realmente bem mais elegante

a resposta que eu encontrei foi b) 22
On Sat, Nov 13, 2004 at 01:54:21PM -0800, Artur Posenato wrote:
 Tente utilizar esta rela??o:
 max(a,b) = (a + b + |b - a|)/2. Talvez tenha uma
 solu??o elegante, mas s? consegui pensar no m?todo da
 for?a bruta (calcular as possibilidades todas).
 
 Artur
 
 --- aryqueirozq [EMAIL PROTECTED] wrote:
 
  Se max(a,b) denota o maior dentre os n?meros reais a
  e b, quantas solu??es inteiras admite a desigualdade
  max(2x+5, 8-3x)35 ?
  
  a) 21   b) 22 c) 23 d) 24   
e) 25
  
  
  Agrade?o desde de j?.
   
 
 __
  Acabe com aquelas janelinhas que pulam na sua tela.
  AntiPop-up UOL - ? gr?tis!
  http://antipopup.uol.com.br/
  
  
 
 
 
   
 __ 
 Do you Yahoo!? 
 Check out the new Yahoo! Front Page. 
 www.yahoo.com 
  
 
 =
 Instru??es para entrar na lista, sair da lista e usar a lista em
 http://www.mat.puc-rio.br/~nicolau/olimp/obm-l.html
 =
=
Instruções para entrar na lista, sair da lista e usar a lista em
http://www.mat.puc-rio.br/~nicolau/olimp/obm-l.html
=


Re: [obm-l] Duvidas II

2004-11-13 Por tôpico Eduardo Henrique Leitner
opa, c) 23

On Sat, Nov 13, 2004 at 08:45:34PM -0200, Eduardo Henrique Leitner wrote:
 pode-se resolver a inequação 2x+5 / 8 - 3x e obter uma solução S1
 resolve-se a inequação 2x+5 = 35 e obtém-se uma solução S2
 
 faz-se a interseção das soluções S1 e S2 e chega-se a uma solução S3
 
 resolve-se a inequação 8 - 3x = 2x + 5 e obtém-se uma solução S4
 resolve-se a inequação 8 - 3x = 35 e obtém-se uma solução S5
 
 faz-se a interseçao das soluçoes S4 e S5 e chega-se a uma solução S6
 
 a solução da questao é dada pela uniao da soluçao S3 com a solução S6
 
 essa foi a primeira maneira que pensei, mas a solução que o Artur apresentou é mais 
 rápida e realmente bem mais elegante
 
 a resposta que eu encontrei foi b) 22
 On Sat, Nov 13, 2004 at 01:54:21PM -0800, Artur Posenato wrote:
  Tente utilizar esta rela??o:
  max(a,b) = (a + b + |b - a|)/2. Talvez tenha uma
  solu??o elegante, mas s? consegui pensar no m?todo da
  for?a bruta (calcular as possibilidades todas).
  
  Artur
  
  --- aryqueirozq [EMAIL PROTECTED] wrote:
  
   Se max(a,b) denota o maior dentre os n?meros reais a
   e b, quantas solu??es inteiras admite a desigualdade
   max(2x+5, 8-3x)35 ?
   
   a) 21   b) 22 c) 23 d) 24   
 e) 25
   
   
   Agrade?o desde de j?.

  
  __
   Acabe com aquelas janelinhas que pulam na sua tela.
   AntiPop-up UOL - ? gr?tis!
   http://antipopup.uol.com.br/
   
   
  
  
  
  
  __ 
  Do you Yahoo!? 
  Check out the new Yahoo! Front Page. 
  www.yahoo.com 
   
  
  =
  Instru??es para entrar na lista, sair da lista e usar a lista em
  http://www.mat.puc-rio.br/~nicolau/olimp/obm-l.html
  =
 =
 Instruções para entrar na lista, sair da lista e usar a lista em
 http://www.mat.puc-rio.br/~nicolau/olimp/obm-l.html
 =
=
Instruções para entrar na lista, sair da lista e usar a lista em
http://www.mat.puc-rio.br/~nicolau/olimp/obm-l.html
=


Re: [obm-l] + duvidas

2004-11-25 Por tôpico Eduardo Henrique Leitner

os divisores postivos serão:

1, p, p^2

logo: 1 + p + p^2 = 31 = p + p^2 = 30

como o valor é pequeno, tu podes chutar algns valores e encontrar p = 5

alternativa A

On Thu, Nov 25, 2004 at 02:44:06AM -0200, aryqueirozq wrote:
 Se p é um número natural primo e a soma de todos os divisores positivos de p2  é 
 igual a 31, então p é igual a :
 A) 5
 B) 7
 C) 3
 D) 2
 E) 11
  
 __
 Acabe com aquelas janelinhas que pulam na sua tela.
 AntiPop-up UOL - É grátis!
 http://antipopup.uol.com.br/
 
=
Instruções para entrar na lista, sair da lista e usar a lista em
http://www.mat.puc-rio.br/~nicolau/olimp/obm-l.html
=


Re: [obm-l] IME!!!!

2004-11-28 Por tôpico Eduardo Henrique Leitner
de acordo com os gabaritos dos cursinhos eu acertei todas

On Sun, Nov 28, 2004 at 03:22:42AM -0200, vinicius wrote:
 AÍ GALERA, ALGUÉM AÍ MANDOU BEM NO IME, TIPO MAIS QUE 8 EM MAT/
 
 =
 Instruções para entrar na lista, sair da lista e usar a lista em
 http://www.mat.puc-rio.br/~nicolau/olimp/obm-l.html
 =
=
Instruções para entrar na lista, sair da lista e usar a lista em
http://www.mat.puc-rio.br/~nicolau/olimp/obm-l.html
=


[obm-l] Cálculo I / Geometria

2004-12-02 Por tôpico Henrique Patrício Sant'Anna Branco



Alguém poderia me dar uma mão com esse? Geometria 
não é meu forte de forma alguma...
Uma mulher em um ponto A na praia de lago circular 
com raio 2 mi quer chegar ao ponto C diametralmente oposto a A do outro lado do 
lado no menor tempo possível. Ela pode andar a uma taxa de 4mi/h e remar um bote 
a 2mi/h. Como ela deve proceder?
Grato,Henrique.


Re: [obm-l] questao do ITA furada

2004-12-15 Por tôpico Eduardo Henrique Leitner
aa, entao deve ser por isso que o anglo ainda nao divulgou a resolucão da questao 
30... eles devem estar tentando considerar que x pode ser complexo...

Questão 30. Determine todos os valores reais de a para os quais a equação

 (x-1)^2 = |x - a|

admita exatamente três soluções distintas.

hehehe, eles devem estar tendo moh trabalhão...


On Wed, Dec 15, 2004 at 07:47:42PM -0200, Claudio Buffara wrote:
 on 15.12.04 19:21, Fabio Niski at [EMAIL PROTECTED] wrote:
 
  A questao 11 do ITA No desenvolvimento de (ax^2 + -2bx + c + 1)^5
  obtem-se um polinomio p(x) cujos coeficientes somam 32. Se 0 e -1 sao
  raizes de p(x), entao a soma a + b + c é igual a
  a) -1/2 b) -1/4 c) 1/2 d)1 e)3/2
  
  Pelo o que eu vi, Etapa, Poliedro e Objetivo marcaram A.
  O Anglo observou corretamente que existem 5 possiveis valores possiveis
  pra soma e a questao deveria ser cancelada.
 
 Essa eh complicada. Nao ha nada no enunciado que diga que a deve ser real,
 apesar dessa ser uma hipotese razoavel.
 
 Qual foi o veredito?
 
 
 
 
 
 =
 Instruções para entrar na lista, sair da lista e usar a lista em
 http://www.mat.puc-rio.br/~nicolau/olimp/obm-l.html
 =
=
Instruções para entrar na lista, sair da lista e usar a lista em
http://www.mat.puc-rio.br/~nicolau/olimp/obm-l.html
=


Re: [obm-l] lim x-+oo sinx/x

2004-12-10 Por tôpico Eduardo Henrique Leitner
oras, tu tens um numerador que varia de -1 a 1 e um denominador que tende ao infinito.

logo, a fraçao tende a zero

On Fri, Dec 10, 2004 at 10:49:15AM -0300, Marcos Victor wrote:
 lim x-+oo sinx/x 
 quando eh esse limite.
 quando x tende a zero é um, mas e esse?
  
  
 
 __
 Converse com seus amigos em tempo real com o Yahoo! Messenger 
 http://br.download.yahoo.com/messenger/ 
=
Instruções para entrar na lista, sair da lista e usar a lista em
http://www.mat.puc-rio.br/~nicolau/olimp/obm-l.html
=


Re: [obm-l] questao do ITA furada

2004-12-15 Por tôpico Eduardo Henrique Leitner
a 26 eu achei particularmente complicada...
mesmo que eu soubesse fazer, nao havia espaço para fazer uma resolução como a que o 
etapa fez...

no geral eu achei que essa prova estava tao ou mais fahcil que a do ano passado... sei 
lah, eu nao mando muito bem em matemática [pelo menos, é o que as olimpiadas mostram) 
e consegui fazer todas (exceto 2 erros tolos em questoes objetivas do tipo: 107/280 = 
0,34...; e achar a semi-distância focal quando foi pedida a própria distancia focal, e 
essa 26 que realmente nao consegui]

nao sei se com essa prova eles conseguiram selecionar muita coisa... tenho certeza de 
que existem MUITAS pessoas que fizeram a prova que sao melhores que eu e devem ter 
ficado com notas parecidas...

On Wed, Dec 15, 2004 at 09:46:43PM -0200, Fabio Niski wrote:
 Eles tb nao divulgaram a 20 e 26.
 Pode ser que seja isso, ou pode ser que o pessoal ta tomando um café.
 
 
 Eduardo Henrique Leitner wrote:
 
 aa, entao deve ser por isso que o anglo ainda nao divulgou a resolucão da 
 questao 30... eles devem estar tentando considerar que x pode ser 
 complexo...
 
 Questão 30. Determine todos os valores reais de a para os quais a equação
 
  (x-1)^2 = |x - a|
 
 admita exatamente três soluções distintas.
 
 hehehe, eles devem estar tendo moh trabalhão...
 
 
 On Wed, Dec 15, 2004 at 07:47:42PM -0200, Claudio Buffara wrote:
 
 on 15.12.04 19:21, Fabio Niski at [EMAIL PROTECTED] wrote:
 
 
 A questao 11 do ITA No desenvolvimento de (ax^2 + -2bx + c + 1)^5
 obtem-se um polinomio p(x) cujos coeficientes somam 32. Se 0 e -1 sao
 raizes de p(x), entao a soma a + b + c é igual a
 a) -1/2 b) -1/4 c) 1/2 d)1 e)3/2
 
 Pelo o que eu vi, Etapa, Poliedro e Objetivo marcaram A.
 O Anglo observou corretamente que existem 5 possiveis valores possiveis
 pra soma e a questao deveria ser cancelada.
 
 
 Essa eh complicada. Nao ha nada no enunciado que diga que a deve ser real,
 apesar dessa ser uma hipotese razoavel.
 
 Qual foi o veredito?
 
 
 
 
 
 =
 Instruções para entrar na lista, sair da lista e usar a lista em
 http://www.mat.puc-rio.br/~nicolau/olimp/obm-l.html
 =
 
 =
 Instruções para entrar na lista, sair da lista e usar a lista em
 http://www.mat.puc-rio.br/~nicolau/olimp/obm-l.html
 =
 
 
 
 =
 Instruções para entrar na lista, sair da lista e usar a lista em
 http://www.mat.puc-rio.br/~nicolau/olimp/obm-l.html
 =
=
Instruções para entrar na lista, sair da lista e usar a lista em
http://www.mat.puc-rio.br/~nicolau/olimp/obm-l.html
=


Re: [obm-l] Re:[obm-l] Pra que serve a matemática?

2004-12-28 Por tôpico Eduardo Henrique Leitner
bom... eu creio que essa pergunta esteja descontextualizada... afinal, a lista 
é para aqueles que já entenderam para que serve a matemática e se divertem com 
problema olímpicos..

mas esta resposta também está descontextualizada da pergunta, afinal, pra que 
diabos alguém iria querer saber quanto vale a soma dos inteiros de 1 até 100

essa resposta lembrou-me daquela frase anedótica sobre computadores: os 
computadores vieram para resolver os problemas que antes deles nao tínhamos.

 sem mais 

On Tue, Dec 28, 2004 at 09:22:21PM -0200, Osvaldo Mello Sponquiado wrote:
 Para ser aplicada no cotidiano, visite o site do IMPA (Instituto de 
 Matemática Pura e Aplicada)
 Aos dez anos de idade Gauss estava em um dia normal de escola quando um 
 professor indagou para a sala toda quanto valia a soma dos inteiros de 1 até 
 100.
 Ele respondeu, em segundos, 5050. Simplesmente notou que 1+100=2+99 e foi 
 agrupando as 100 parcelas em 50 grupos de mesma soma (101), logo Soma=50.101
 por exemplo.
 
 
 
  Boa tarde
 
  Pra que serve a matemática?
 
  Pergunta um tanto óbvia, mas quando pensamos que algo é muito óbvio, é
  quando não estamos pensando.
 
  = 
  Instruções para entrar na lista, sair da lista e usar a lista em
  http://www.mat.puc-rio.br/~nicolau/olimp/obm-l.html
  = 
 
 Atenciosamente,
 Osvaldo Mello Sponquiado
 Engenharia Elétrica, 2ºano
 UNESP - Ilha Solteira 
  
 __
 Acabe com aquelas janelinhas que pulam na sua tela.
 AntiPop-up UOL - É grátis!
 http://antipopup.uol.com.br/
 
=
Instruções para entrar na lista, sair da lista e usar a lista em
http://www.mat.puc-rio.br/~nicolau/olimp/obm-l.html
=


[obm-l] Probabilidade

2005-01-08 Por tôpico Henrique Patrício Sant'Anna Branco
Pessoal, tô empacado com esses aqui. Se alguém puder me indicar um caminho,
fico agradecido.

1 - Suponha que uma caixa contém 3 bolas numeradas de 1 a 3. Seleciona-se
sem reposição duas bolas da caixa. Seja X o número da primeira bola e Y o
número da segunda bola.
Determinar a covariância e o coeficiente de associação linear entre X e Y.

2 - Seja X ~ Exp(a) e m um inteiro não-negativo. Define-se Y por Y = m se m
= X  m + 1. Como se distribui Y?

Grato,
Henrique.



-- 

Checked by AVG Anti-Virus.
Version: 7.0.300 / Virus Database: 265.6.9 - Release Date: 6/1/2005

=
Instruções para entrar na lista, sair da lista e usar a lista em
http://www.mat.puc-rio.br/~nicolau/olimp/obm-l.html
=


Re: [obm-l] oi!

2005-01-09 Por tôpico Eduardo Henrique Leitner
aee, se tu fores bonita vai pro ITA! daí nos conhecemos lá! ;)

desculpem, nao resisti

On Sun, Jan 09, 2005 at 05:50:22PM -0200, Kellem :-) 100% SeJ wrote:
 
 Oi Gente
 Bem, meu nome é Kellem e sou do RJ. Fazia UFF e, agora q passeo pro IME e
 pro ITA, vou pra um dos dois, só q, até agora, fico sem dormir sem saber o q
 fazer, pq tb faço Iniciação Científica no IMPA! Eu não sei se fico pra
 arriscar o IMPA... O problema é q gosto muito de matemática, mas acho q não
 sei mto não (sempre acho q não sei nada e q vou tirar zero). Enfim, tô
 aqui, e espero poder ajudar e obter ajuda nos probleminhas, tá?
 Alguém daqui me conhece (ou eu conheço??)?
 BJão
 Kellem
 
 =
 Instruções para entrar na lista, sair da lista e usar a lista em
 http://www.mat.puc-rio.br/~nicolau/olimp/obm-l.html
 =
=
Instruções para entrar na lista, sair da lista e usar a lista em
http://www.mat.puc-rio.br/~nicolau/olimp/obm-l.html
=


[obm-l] Probabilidade

2005-01-21 Por tôpico Henrique Patrício Sant'Anna Branco
Alguém pode ajudar nesses dois?
O número dois até consigo resolver a primeira parte (achar a distribuição de
X, geométrica), mas não consigo montar a segunda parte.

1. Suponha que os tempos que dois estudantes levam para resolver um problema
sao independentes e se distribuem exponencialmente com parâmetro a.
Determine a probabilidade de que o primeiro estudante necessite pelo menos
do dobro do tempo gasto pelo segundo estudante para resolver o problema.

2. Suponha que X tem uma densidade exponencial de parâmetro a e que
X_epsilon seja defeinido em termos de X e epsilon  0 por X_epsilon =
epsilon*k se epsilon*k = X  epsilon(k+1) para k inteiro. Qual a
distribuição de (E_epsilon)/(epsilon)? Obtenha E(X_epsilon) e determine seu
limite quando epsilon - 0.

Grato,
Henrique.



-- 

Checked by AVG Anti-Virus.
Version: 7.0.300 / Virus Database: 265.7.1 - Release Date: 19/1/2005

=
Instruções para entrar na lista, sair da lista e usar a lista em
http://www.mat.puc-rio.br/~nicolau/olimp/obm-l.html
=


Re: [obm-l] Probabilidade

2005-01-22 Por tôpico Henrique Patrício Sant'Anna Branco
 1. Suponha que os tempos que dois estudantes levam para resolver um
problema
 sao independentes e se distribuem exponencialmente com parâmetro a.
 Determine a probabilidade de que o primeiro estudante necessite pelo
menos
 do dobro do tempo gasto pelo segundo estudante para resolver o problema.

 Acho que esse é só uma questão de expressar a probabilidade em termos de
 probab. condicional...

 Condicione no tempo que o segundo estudante leva para resolver o problema.
 Seja A o evento desejado (estudante 1 leva pelo menos o dobro do tempo
 do estudante 2).
 Seja X a variável aleatória exponencial de param. a correspondente ao 1º
 estudande e Y a do segundo.
 Seja f a função densidade de uma var. exp. de parâmetro a
 Pr[A] = Integral_{0, +oo} Pr[X = 2y | Y = y]*f(y) dy = Integral_{0,
 +oo} Pr[X = 2y]*f(y) dy já que X e Y são independentes.

O meu problema tem sido exatamente calcular P(X = 2y). De qualquer forma,
consigo a resposta a/3 (assumi que os parâmetros sao iguais pras duas
distribuições, já que o problema deixa isso meio implícito). Mas a resposta
certa é 1/3.

O que poso fazer?

Grato,
Henrique.



-- 

Checked by AVG Anti-Virus.
Version: 7.0.300 / Virus Database: 265.7.2 - Release Date: 21/1/2005

=
Instruções para entrar na lista, sair da lista e usar a lista em
http://www.mat.puc-rio.br/~nicolau/olimp/obm-l.html
=


Re: [obm-l] Probabilidade

2005-01-22 Por tôpico Henrique Patrício Sant'Anna Branco
Acabei conseguindo resolver...
Mas não entendi uma parte da sua resolução, segue abaixo:

Pr[A] = Integral_{0, +oo} Pr[X = 2y | Y = y]*f(y) dy = Integral_{0,
+oo} Pr[X = 2y]*f(y) dy

A primeira integral não deveria ser simplesmente Integral_{0, +oo} Pr[X =
2y | Y = y], que se transforma na segunda pela independência?

Muito obrigado mesmo.
Henrique.

- Original Message - 
From: Domingos Jr. [EMAIL PROTECTED]
To: obm-l@mat.puc-rio.br
Sent: Saturday, January 22, 2005 3:59 PM
Subject: Re: [obm-l] Probabilidade


 Henrique Patrício Sant'Anna Branco wrote:

 Alguém pode ajudar nesses dois?
 O número dois até consigo resolver a primeira parte (achar a distribuição
de
 X, geométrica), mas não consigo montar a segunda parte.
 
 1. Suponha que os tempos que dois estudantes levam para resolver um
problema
 sao independentes e se distribuem exponencialmente com parâmetro a.
 Determine a probabilidade de que o primeiro estudante necessite pelo
menos
 do dobro do tempo gasto pelo segundo estudante para resolver o problema.
 
 

 Acho que esse é só uma questão de expressar a probabilidade em termos de
 probab. condicional...

 Condicione no tempo que o segundo estudante leva para resolver o problema.
 Seja A o evento desejado (estudante 1 leva pelo menos o dobro do tempo
 do estudante 2).
 Seja X a variável aleatória exponencial de param. a correspondente ao 1º
 estudande e Y a do segundo.
 Seja f a função densidade de uma var. exp. de parâmetro a
 Pr[A] = Integral_{0, +oo} Pr[X = 2y | Y = y]*f(y) dy = Integral_{0,
 +oo} Pr[X = 2y]*f(y) dy já que X e Y são independentes.


 Abraços.
 =
 Instruções para entrar na lista, sair da lista e usar a lista em
 http://www.mat.puc-rio.br/~nicolau/olimp/obm-l.html
 =



 -- 

 Checked by AVG Anti-Virus.
 Version: 7.0.300 / Virus Database: 265.7.2 - Release Date: 21/1/2005





-- 

Checked by AVG Anti-Virus.
Version: 7.0.300 / Virus Database: 265.7.2 - Release Date: 21/1/2005

=
Instruções para entrar na lista, sair da lista e usar a lista em
http://www.mat.puc-rio.br/~nicolau/olimp/obm-l.html
=


[obm-l] Problema interessante em MATLAB

2005-02-24 Por tôpico Henrique Lima Santana

Olá pessoal, gostaria da ajuda de vocês nesse problema (de médias móveis 
simples) em MATLAB
Bom, o problema consiste em bolar o algoritmo para cálculo da média móvel e, 
em seguida, montar um gráfico, o problema é que comecei agora em linguagem 
de programação...não sei fazer direito:

A média móvel simples é calculada a partir de r observações, conhecendo-se 
valores z(i)  (que são variáveis para cada amostra a partir das quais 
deseja-se fazer a previsão):
M(t)=[sum z(i)] / r , com i=t,t-1,...,t-r+1 (r valores) ; onde t é o período 
de oservação. Para t, por exemplo, igual à 120 e r=7 , pede-se calcular 
M(7),M(8),...,M(120) e fazer o gráfico M(t)xt a partir de valores 
z(1),z(2),...,z(120) conhecidos.

Bom, tentei fazer o algoritmo:
t=input (´coloque o valor de t=período´)  (ps.: no problema onde estava 
testando, t era 120 dias)
r=input (´coloque o valor de r=nº de observações´)
for k=linspace(1,t,t)
soma=0
for i=linspace(1,r,r)
indice=(k+i-1)
soma=z(indice) +soma
end
M(k+i-1)=soma/r
plot(M(k+i-1),t)
clear t,r,k,i,indice

Bom, não estou conseguindo fazer o gráfico...afora que nem sei se isso que 
eu fiz, de fato, calcula os M(h) valores (h=7,...,120)
Outra coisa, tô sem o MATLAB aqui em casa, desse jeito que eu fiz o programa 
vai pedir os t valores do z ?

Agradeço qualquer ajuda!
[]´s
Henrique
_
MSN Messenger: converse online com seus amigos .  
http://messenger.msn.com.br

=
Instruções para entrar na lista, sair da lista e usar a lista em
http://www.mat.puc-rio.br/~nicolau/olimp/obm-l.html
=


[obm-l] Problema interessante em MATLAB

2005-02-26 Por tôpico Henrique Lima Santana

Olá pessoal, gostaria da ajuda de vocês nesse problema (de médias móveis 
simples) em MATLAB
Bom, o problema consiste em bolar o algoritmo para cálculo da média móvel 
e, em seguida, montar um gráfico, o problema é que comecei agora em 
linguagem de programação...não sei fazer direito:

A média móvel simples é calculada a partir de r observações, conhecendo-se 
valores z(i)  (que são variáveis para cada amostra a partir das quais 
deseja-se fazer a previsão):
M(t)=[sum z(i)] / r , com i=t,t-1,...,t-r+1 (r valores) ; onde t é o 
período de oservação. Para t, por exemplo, igual à 120 e r=7 , pede-se 
calcular M(7),M(8),...,M(120) e fazer o gráfico M(t)xt a partir de valores 
z(1),z(2),...,z(120) conhecidos.

Bom, tentei fazer o algoritmo:
t=input (´coloque o valor de t=período´)  (ps.: no problema onde estava 
testando, t era 120 dias)
r=input (´coloque o valor de r=nº de observações´)
for k=linspace(1,t,t)
soma=0
for i=linspace(1,r,r)
indice=(k+i-1)
soma=z(indice) +soma
end
M(k+i-1)=soma/r
plot(M(k+i-1),t)
clear t,r,k,i,indice

Bom, não estou conseguindo fazer o gráfico...afora que nem sei se isso que 
eu fiz, de fato, calcula os M(h) valores (h=7,...,120)
Outra coisa, tô sem o MATLAB aqui em casa, desse jeito que eu fiz o 
programa vai pedir os t valores do z ?

Agradeço qualquer ajuda!
[]´s
Henrique
_
MSN Messenger: converse online com seus amigos .  
http://messenger.msn.com.br

=
Instruções para entrar na lista, sair da lista e usar a lista em
http://www.mat.puc-rio.br/~nicolau/olimp/obm-l.html
=
_
MSN Messenger: converse online com seus amigos .  
http://messenger.msn.com.br

=
Instruções para entrar na lista, sair da lista e usar a lista em
http://www.mat.puc-rio.br/~nicolau/olimp/obm-l.html
=


[obm-l] NP complexo

2005-03-13 Por tôpico Henrique Lima Santana

Oi pessoal, tudo bem?
Bom, gostaria, se possível, da ajuda de vcs nesse problema (probabilidades):
Tem-se n áreas; deve-se, primeiramente, particionar n de todas as maneiras 
possíveis:
(a) (1,1,...,1) n 1´s
(b) (2,1,...,1) (n-2) 1´s
 .
 .
 .
(c)   (n)
E assim, contar as possibilidades, que seriam (de (a) , por exemplo) : 1 com 
1 com 1...com 1 ou A,B,... (onde A,B,...seriam as n áreas) . Total de 
possibilidades=1.  De (b) seria : 2 com 1 com 1... com 1 , ou seja : 
AB,C,D,... ; AC,B,D,... : total de possibilidades: ?. A (c) seria 
ABC Total de possibilidades = 1
Façamos um caso de n pequeno : n=3
Teríamos:
*  (1,1,1) : 1 com 1 com 1 : A,B,C (sendo A,B e C as áreas) , total de  
possibilidades = 1 (como poderíamos calcular essa probabilidade? C3,0 ?)
*   (2,1) : 2 com 1 : AB,C ; AC,B e BC,A  . Total de possibilidades: 3 
(seria C3,2 * C1,1 ?? )
*   (3) : ABC . Total de possibilidades : 1 (C3,3 ?)
Assim, pro n=3 se teria : C3,0 + C3,2*C1,1 + C3,3 =5

É basicamente isso, como eu poderia generalizar o caso prum n qualquer??
Ainda tem um detalhe: essas seriam as possibilidades totais, mas precisa-se 
das possibilidades reais , i.e., apenas as possibilidades em que tivermos 
áreas adjacentes seriam válidas. As áreas seriam agrupadas de duas em duas 
(uma área urbana e outra rural de uma cidade qualquer, onde a área urbana 
fica dentro da rural --  poderíamos considerar 2 circunferência, uma dentro 
da outra) . Por exemplo, por caso n=3 consideraríamos  a cidade X com área 
urbana= A e área rural = B . E a cidade Y com área urbana =C . Assim, a 
possibilidade AC,B seria inválida já que A  e C não são adjacentes...assim, 
as possibilidades totais seriam 5 mas as reais seriam 4 ...
É isso, deu pra entender ? q:-]
Muito obrigado pela atenção dispensada.
[]´s
Henrique

_
MSN Messenger: converse online com seus amigos .  
http://messenger.msn.com.br

=
Instruções para entrar na lista, sair da lista e usar a lista em
http://www.mat.puc-rio.br/~nicolau/olimp/obm-l.html
=


[obm-l] Cálculo de Probabilidades e Teoria da Medida

2005-03-30 Por tôpico Henrique Patrício Sant'Anna Branco
Pessoal,

Estou estudando Cálculo de Probabilidades de uma visão um pouco mais
avançada, ao ponto de despertar minha curiosidade sobre a Teoria da Medida.

Alguém pode me indicar bons livros/sites para pesquisa, bem como os
pré-requisitos pra estudar o assunto?

Grato,
Henrique.

=
Instruções para entrar na lista, sair da lista e usar a lista em
http://www.mat.puc-rio.br/~nicolau/olimp/obm-l.html
=


[obm-l] Variância de sigma^2

2005-04-24 Por tôpico Henrique Patrício Sant'Anna Branco
Pessoal,

Preciso de ajuda nesse.

Sendo X_1, ..., X_n uma amostra aleatória de uma N(mu, tau^2), calcule
Var(sigma^2), onde sigma^2 = (1/n * Sum_{i = 1}^n (X_i - Xbarra) é o
estimador de máxima verossimilhança da variância.

N(mu, tau^2) é a distribuição normal de média mu e variância tau^2.

Sei que dá pra fazer Var(sigma^2) = Var((n-1)/n S^2) = ((n-1)/n)^2 Var(S^2),
mas não consigo calcular nem Var(S^2).

Alguém pode dar uma luz?

Grato,
Henrique.

=
Instruções para entrar na lista, sair da lista e usar a lista em
http://www.mat.puc-rio.br/~nicolau/olimp/obm-l.html
=


[obm-l] Variância de s^2

2005-05-17 Por tôpico Henrique Patrício Sant'Anna Branco
Pessoal,

Fiz uma pergunta nessa lista há um tempo sobre como calcular a Var(s^2),
onde s^2 = 1/(n-1) Sum_i^n (x_i - xbarra) e os x_i são amostras aleatórias
da Normal com média a e variância b.

Consegui resolver esse problema hoje (é bem simples até). Segue a resposta,
a quem possa interessar:

Podemos escrever (usando as propriedades da variância)

Var(s^2) = b^2/(n-1)^2 * Var((n-1)s^2/b)

(n-1)s^2/b = v tem distribuição qui-quadrado com n - 1 graus de liberdade.
Logo, Var(v) = 2(n-1). Fazendo as continhas, chegamos a Var(s^2) =
2b^2/(n-1).

Agradeço aos que responderam.

Henrique.

=
Instruções para entrar na lista, sair da lista e usar a lista em
http://www.mat.puc-rio.br/~nicolau/olimp/obm-l.html
=


[obm-l] k-ésima derivada.

2005-05-26 Por tôpico Henrique Patrício Sant'Anna Branco
Alguém pode me indicar como cálcular a k-ésima derivada de a/(a - it) em
relação a t, ou seja, a fórmula geral da derivada?

Grato,
Henrique.

=
Instruções para entrar na lista, sair da lista e usar a lista em
http://www.mat.puc-rio.br/~nicolau/olimp/obm-l.html
=


[obm-l] Questão simples

2006-03-24 Por tôpico Henrique Patrício Sant'Anna Branco

Pessoal,

Esta é uma questão bem simples, mas gostaria que os colegas me indicassem a 
maneira mais didática de ensiná-la a um aluno sem muita prática com 
matemática.


Curiosamente, dois atendentes de um banco observaram que, durante o 
expediente bancário, o número de clientes que cada um havia atendido era 
inversamente proporcional às suas respectivas idades, 36 e 38 anos. Se um 
deles atendeu 4 clientes a mais que o outro, então o total de pessoas 
atendidas pelo mais velho foi...?


A resposta é 12.

Grato,
Henrique. 


=
Instruções para entrar na lista, sair da lista e usar a lista em
http://www.mat.puc-rio.br/~nicolau/olimp/obm-l.html
=


[obm-l] Re: [obm-l] Questão simples

2006-03-24 Por tôpico Henrique Patrício Sant'Anna Branco

Retificando...

Curiosamente, dois atendentes de um banco observaram que, durante o
expediente bancário, o número de clientes que cada um havia atendido era
inversamente proporcional às suas respectivas idades, 36 e 48 anos. Se um
deles atendeu 4 clientes a mais que o outro, então o total de pessoas
atendidas pelo mais velho foi...?

- Original Message - 
From: Henrique Patrício Sant'Anna Branco [EMAIL PROTECTED]

To: obm-l@mat.puc-rio.br
Sent: Friday, March 24, 2006 2:15 PM
Subject: [obm-l] Questão simples



Pessoal,

Esta é uma questão bem simples, mas gostaria que os colegas me indicassem 
a maneira mais didática de ensiná-la a um aluno sem muita prática com 
matemática.


Curiosamente, dois atendentes de um banco observaram que, durante o 
expediente bancário, o número de clientes que cada um havia atendido era 
inversamente proporcional às suas respectivas idades, 36 e 38 anos. Se um 
deles atendeu 4 clientes a mais que o outro, então o total de pessoas 
atendidas pelo mais velho foi...?


A resposta é 12.

Grato,
Henrique.
=
Instruções para entrar na lista, sair da lista e usar a lista em
http://www.mat.puc-rio.br/~nicolau/olimp/obm-l.html
=



=
Instruções para entrar na lista, sair da lista e usar a lista em
http://www.mat.puc-rio.br/~nicolau/olimp/obm-l.html
=


[obm-l] Re: Questão simples

2006-03-27 Por tôpico Henrique Patrício Sant'Anna Branco

Leonardo,

Essa era mais ou menos a explicação que tinha em mente. Pensei que, para 
alguém não habituado ao pensamento matemático, esse raciocínio fosse de 
alguma forma complicado.


Muito obrigado pelo retorno.
Henrique.

- Original Message - 
From: Leonardo de Almeida Matos Moraes [EMAIL PROTECTED]

To: obm-l@mat.puc-rio.br
Sent: Friday, March 24, 2006 3:16 PM
Subject: [obm-l] RES: [obm-l] Re: [obm-l] Questão simples



Henrique,

no meu ponto de vista, acho facil que voce explique, primeiramente, o que 
e'

ser inversamente proporcional. Chamando de N_a o numero de pessoas que o
atendente de 36 anos atendeu e N_b o numero de pessoas que o atendente de 
48

anos atendeu,

N_a = k * 1/36 e N_b = k * 1/48

Como voce sabe que quem atende mais pessoas e' o de menor idade, ja' que
estes numeros sao inversamente proporcionais, quem atendeu mais pessoas 
foi

o atendente de 36 anos e:

N_a = N_b + 4

de onde voce tira k = 12*48

Jogando este valor na formula de N_b, voce encontra N_b = 12 atendimentos.

Espero ter ajudado.

Abracos,

Leonardo.

=
Instruções para entrar na lista, sair da lista e usar a lista em
http://www.mat.puc-rio.br/~nicolau/olimp/obm-l.html
=


__ Informação do NOD32 IMON 1.1457 (20060324) __

Esta mensagem foi verificada pelo NOD32 sistema antivírus
http://www.eset.com.br




=
Instruções para entrar na lista, sair da lista e usar a lista em
http://www.mat.puc-rio.br/~nicolau/olimp/obm-l.html
=


[obm-l] Experiências com plano inclinado (off-topic)

2006-10-16 Por tôpico Henrique Patrício Sant'Anna Branco

Pessoal,

Sei que é uma lista de matemática, mas estou com um problema em um 
experimento bem simples.


Estou fazendo uma experiência com plano inclinado da seguinte forma: 
utilizando-se um trilho de ar, coloco um corpo para percorrer 10 intervalos 
de 10cm cada um e meço o tempo que levou cada etapa.
O corpo tem uma pequena aleta, de largura L, medida com um paquímetro. A 
cada 10 intervalos de 10cm, meço o tempo que levou para a aleta passar pelo 
sensor (um tempo ínfimo, é claro).


O objetivo é determinar a aceleração da gravidade local. Após medir tudo (e 
conferir), segui os seguintes procedimentos:
1. Somei o comprimento da aleta aos deslocamentos sucessivos, obtendo 
10.514cm, 20.514cm, ..., 100.514cm.
2. Somei o tempo que o corpo leva para percorrer cada um dos intervalos 
(10cm, 20cm etc) ao tempo que levou para a aleta passar pelo sensor em cada 
um destes intervalos (note-se que o corpo sempre parte do repouso do topo do 
plano, ou seja, t = 0 e x = 0).
3. Agora é a parte que pega... Para fazer g = a*sen(theta), calculo a 
aceleração (da forma como procedi, creio que chego à aceleração instantanea, 
certo?) pelas três formulas conhecidas da cinematica:

  i. v = a*t
  ii. x = (a*t^2)/2
  iii. v^2 = 2*a*x
  Na verdade, estou utilizando Mínimos Quadrados para ajustar estas 
equações aos dados, obtendo o valor de a, que é o unico desconhecido.
  O problema é: utilizando a equação i., obtenho a = 10.814, o que me dá g 
= 981.24 (muito bom, por sinal). Mas por ii. obtenho a = 21.934 e, por iii., 
a = 5.4, tudo em cm/s^2.
  Tenho certeza que meu ajuste está correto. Note-se as medidas são 
múltiplas (aproximadas) do valor correto, a = 10.(algumacoisa).

  O que pode estar ocorrendo? Alguma idéia?

Obrigado,
Henrique. 


=
Instruções para entrar na lista, sair da lista e usar a lista em
http://www.mat.puc-rio.br/~nicolau/olimp/obm-l.html
=


[obm-l] Um problema

2006-12-06 Por tôpico Paulo Henrique Souza Lima
Oi pessoal,
 
Um problema:
Prove que

\sum_{n=0}^i C_{i,n} (-1)^n (k-i)/ (n+k-i) = 1/C_{k,i},
 
para ki.
 
Este problema surgiu dentro de um problema de probabilidade. Algumas contas no 
computador sugerem que resultado está certo.
 
Obrigado,
Paulo



O Yahoo! está de cara nova. Venha conferir!



___ 
Yahoo! Acesso Grátis - Internet rápida e grátis. Instale 
o discador agora! 
http://br.acesso.yahoo.com

[obm-l] Res: [obm-l] Re: [obm-l] TRANSAÇÕES FALACI OSAS!

2006-12-07 Por tôpico Paulo Henrique Souza Lima
Oi Marcelo,

Concordo com voce nas questoes 1 e 2. Na questao 1, outra forma de ver é que no 
final das contas o livreiro ficou com uma nota de $100 falsa, entao o prejuizo 
dele é $100.

Veja o meu raciocinio na questao 3:
Se todo o dinheiro for verdadeiro, gastei $70 e ganhei $80: lucro $10. 
Portanto, deixei de ganhar $10. 

[]'s,
Paulo



- Mensagem original 
De: Marcelo Salhab Brogliato [EMAIL PROTECTED]
Para: obm-l@mat.puc-rio.br
Enviadas: Quarta-feira, 6 de Dezembro de 2006 18:49:57
Assunto: [obm-l] Re: [obm-l] TRANSAÇÕES FALACIOSAS!


Olá,

1) Inicialmente, o livreiro nao tem prejuizo, que é todo do jornaleiro.. 
pois ele esta com 20 reais verdadeiros.
Mas, o jornaleiro exige que ele devolva os 50 reais, logo, ele fica com o 
prejuizo de 50 reais, 30 do dinheiro + 20 do livro.
Entao, apos a ultima compra, ele fica com um prejuizo de 100 reais, que sao 
50 reais do dinheiro + 50 reais do livro.
nossa.. confusa! hehe Prejuizo = 100 reais!


2) vamos supor que ele tem X reais no comeco.. entao ele compra fica com 
X-30, vende, fica com X-30+40 =X+10,
compra novamente e fica com X+10-50 = X-40
ai ele vende por 60, mas nao recebe o dinheiro, entao, ele fica com X-40, 
logo, o prejuizo é de 40 reais!


3) na primeira compra, eu pagaria 70, mas como usei dinheiro falso, ganhei 
60 reais, que é o preco de mercado do objeto..
entao, eu vendi por 80 reais, entao, eu ganharia no total: 240 reais.. logo, 
deixei de ganhar 240 reais!

bom, aguardo a correcao!

abraços,
Salhab



- Original Message - 
From: Jorge Luis Rodrigues e Silva Luis [EMAIL PROTECTED]
To: obm-l@mat.puc-rio.br
Sent: Wednesday, December 06, 2006 10:41 AM
Subject: [obm-l] TRANSAÇÕES FALACIOSAS!


 PASMEM! O curioso destas situações abaixo é que apesar de girarem em torno 
 de um mesmo assunto, dificilmente alguém consegue acertar ao menos um e 
 raramente os três...

 Um cliente compra um livro que custa 20 reais, e paga com uma nota falsa 
 de 50 reais. Sem troco o livreiro vai até a banca de jornais e troca a 
 nota liberando o cliente. Em seguida entra o jornaleiro exigindo a troca 
 da nota falsa por outra verdadeira. Depois, outro cliente compra um livro 
 que custa 50 reais, e paga com uma nota falsa de 100 reais recebendo como 
 troco a nota falsa de 50 reais. Afinal! Qual o prejuízo do livreiro?

 Um comerciante comprou uma cadeira por $30, vendeu-a logo em sequida por 
 $40, comprou-a de volta por $50 e vendeu-a novamente por $60. Qual o 
 prejuízo do comerciante, já que o cheque de $60 voltou por insuficiência 
 de fundos?

 Comprei um objeto por 70 reais, cujo preço de mercado é 60 reais. Se 
 conseguir vendê-lo por 80 reais, quanto deixarei de ganhar se todo o 
 dinheiro transacionado era falso?   (CAMPEÃO!)

 Divirtam-se e Boas Surpresas!

 _
 Insta-le já o Windows Live Messenger. A nova geração do messenger. 
 http://get.live.com/messenger/overview

 =
 Instruções para entrar na lista, sair da lista e usar a lista em
 http://www.mat.puc-rio.br/~nicolau/olimp/obm-l.html
 =


 -- 
 No virus found in this incoming message.
 Checked by AVG Free Edition.
 Version: 7.1.409 / Virus Database: 268.15.3/562 - Release Date: 1/12/2006

 

=
Instruções para entrar na lista, sair da lista e usar a lista em
http://www.mat.puc-rio.br/~nicolau/olimp/obm-l.html
=








___ 
Você quer respostas para suas perguntas? Ou você sabe muito e quer compartilhar 
seu conhecimento? Experimente o Yahoo! Respostas !
http://br.answers.yahoo.com/

Res: [obm-l] Um problema

2006-12-07 Por tôpico Paulo Henrique Souza Lima
Oi pessoal,

Se eu tenho U_1,U_2,...,U_{k-i} variáveis aleatórias (v.a.) uniformes no 
intervalo [0,1] e T_1,T_2,...,T_i, v.a. uniformes no intervalo [-1,0] todas 
independentes, o evento de interesse (chamamos de E) é aquele em que a 
distancia entre todos os pontos é menor do que 1.

Para isto, basta que a distancia entre o M_1=max_ j \in{1,...,k-i} U_j e 
M_2=min_j \in{1,...,i} T_j seja menor do que 1. Observe que estas v.a. sao 
independentes e que 

P[M_1u] = u^(k-i), para 0u1. 
P[M_2v] = 1-(1-v)^j, para -1v0. 

Entao, fazendo uma transformacao simples, temos:

P[E] = \int_0^1 \int_0^{1-u} i.v^{i-1} . (k-i) . u^{k-i-1} dvdu.

Fazendo um pouco de contas, finalmente, chega-se que

P[E] = \sum_{n=0}^i C_{i,n} (-1)^n (k-i)/ (n+k-i) = 1/C_{k,i},

Que é o que eu pedi para voces. Curiosamente tem uma forma fechada bonitinha.  
Nao conheco nenhum argumento combinatorio para a resolucao do problema, mas com 
um pouco de imaginacao acho que seja possivel obter. 

Muito obrigado,

Paulo



- Mensagem original 
De: Luís Lopes [EMAIL PROTECTED]
Para: obm-l@mat.puc-rio.br
Enviadas: Quinta-feira, 7 de Dezembro de 2006 10:31:22
Assunto: RE: [obm-l] Um problema


Sauda,c~oes,

Oi Ph,

O resultado vale para i=0 (a soma é igual a 1).
Vamos então considerar ki0.

Usando o resultado
\sum_{n=0}^i \binom{i}{n} (-1)^n = 0 (i0)
o resultado a provar é
\sum_{n=0}^i \binom{i}{n} (-1)^{n+1} \frac{n}{n+k-i} .

Vou mudar a notação para uma mais padrão
e provar que

S_n(m) = \sum_{k=0}^n \binom{n}{k} (-1)^{k+1} \frac{k}{k+m-n} =
= \frac{1} {\binom{m}{n}} (mn0).

S_n(m)=\sum_{k\geq0} n \binom{n-1}{k} (-1)^k \frac{1}{k+1+m-n}
pois \binom{n}{k}=\frac{n}{k} \binom{n-1}{k-1}.

S_n(m)=\sum_{k\geq0} t_k, onde
t_k = n \binom{n-1}{k} (-1)^k \frac{1}{k+1+m-n}.

Então t_0=\frac{n}{m+1-n} e \frac{t_{k+1}}{t_k}=
=\frac {(k+m+1-n)(k-n+1)}{(k+m+2-n)(k+1)} .

Um resultado devido a Gauss (séries hipergeométricas)
diz que S_n(m) = t_0 \frac{\Gamma(c)\Gamma(c-a-b)}
{\Gamma(c-a)\Gamma(c-b)}

com a=m+1-n ; b=-n+1 ; c=m+2-n .

Sabendo que \Gamma(p+1)=p! (p inteiro) e fazendo as
contas, vem:

S_n(m) = \frac{n}{m+1-n} \frac{(m+1-n)!(n-1)!}{m!} =
=\frac{n!(m-n)!}{m!}=\frac{1}{\binom{m}{n}}, (mn\geq0) \qed

Qual a interpretação combinatória do resultado (o Claudio iria
certamente perguntar)?

[]'s
Luís

From: Paulo Henrique Souza Lima [EMAIL PROTECTED]
Reply-To: obm-l@mat.puc-rio.br
To: obm lista obm-l@mat.puc-rio.br
Subject: [obm-l] Um problema
Date: Wed, 6 Dec 2006 08:15:35 -0800 (PST)

Oi pessoal,

Um problema:
Prove que

\sum_{n=0}^i C_{i,n} (-1)^n (k-i)/ (n+k-i) = 1/C_{k,i},

para ki.

Este problema surgiu dentro de um problema de probabilidade. Algumas contas 
no computador sugerem que resultado está certo.

Obrigado,
Paulo

_
MSN Busca: fácil, rápido, direto ao ponto.  http://search.msn.com.br

=
Instruções para entrar na lista, sair da lista e usar a lista em
http://www.mat.puc-rio.br/~nicolau/olimp/obm-l.html
=








___ 
Você quer respostas para suas perguntas? Ou você sabe muito e quer compartilhar 
seu conhecimento? Experimente o Yahoo! Respostas !
http://br.answers.yahoo.com/

Res: [obm-l] Notacao na Lista de Discussao

2007-02-15 Por tôpico Paulo Henrique Souza Lima
Oi Pessoal,

Antes de mais nada, queria dizer que sou novo na lista e estou gostando muito 
das mensagens que tenho recebido.

Aproveitando a mensagem do Nicolau e a minha dificuldade em entender algumas 
notacoes matematicas da lista, gostaria de saber se voces sentem a necessidade 
de padronizar uma notacao matematica para a lista.

A maioria de voces conhece o latex? E' um compilador de textos matematicos 
padrao na area cientifica. O que vcs acham? 

Poderiamos criar uma pagina com instrucoes sobre o padrao de notacao usado na 
lista para ser usado tanto para os atuais quanto para os novos usuarios.

Obrigado,
Paulo


- Mensagem original 
De: Nicolau C. Saldanha [EMAIL PROTECTED]
Para: obm-l@mat.puc-rio.br
Enviadas: Quinta-feira, 15 de Fevereiro de 2007 13:58:31
Assunto: [obm-l] Sequencias (era: Ajuda urgente)




Comentários menores: eu não considero o uso de chaves {} adequado
para uma seqüência, chaves para mim são para conjuntos. E tente
usar subjects mais informativos.

[]s, N.
=
Instruções para entrar na lista, sair da lista e usar a lista em
http://www.mat.puc-rio.br/~nicolau/olimp/obm-l.html
=

__
Fale com seus amigos  de graça com o novo Yahoo! Messenger 
http://br.messenger.yahoo.com/ 

Re [obm-l] Provando uma igualdade

2007-05-15 Por tôpico Paulo Henrique Souza Lima
Acho que achei um argumento combinatorio:
 
n! - numero de formas de arranjar os numeros 2,3,...,n+1 (dos n+1 sem utilizar 
o 1).

A seguir, inseriremos o 1 entre os numeros do arranjamento acima: Se nao 
colocarmos o 1 na primeira posicao, temos n possibilidades, dai resulta

n.n! possibilidades de arranjarmos os n+1 numeros sem colocar o 1 na primeira 
posicao.

Fixe o 1 na primeira posicao. Existem (n-1)! possibilidades sem utilizar o 
numero 2. 

A seguir, inseriremos o 2:  Se nao colocarmos o 2 na segunda posicao, temos n-1 
possibilidades, dai resulta
(n-1). (n-1)!

Seguindo com este argumento,

Se fixarmos os numeros o 1,2,...,k nas posicoes 1,2,...,k, e nao utilizarmos o 
k+1, temos (n+1-k)! possibilidades.

A seguir, inseriremos o k+1: Se nao colocarmos o k+1 na posicao k+1, temos 
n+1-k possibilidades, dai resulta
(n+1-k). (n+1-k)! possibilidades,  k=1,...n 

Desta forma, contaremos todas as permutacoes de n+1 numeros, menos aquela em 
que os numeros estao em suas posicoes: 1,2,...,n, n+1

Portanto, (n+1)! -1possibilidades

[]'s,
Paulo

__
Fale com seus amigos  de graça com o novo Yahoo! Messenger 
http://br.messenger.yahoo.com/ 

<    3   4   5   6   7   8   9   >